You are on page 1of 90

Issue No.

1 Page 122 of 289


ST. THERESE- MTC COLLEGES SEAMANSHIP 6
Iloilo, Philippines (Advance Trim, Stability and Stress)
STUDENT LEARNING MODULE

E
Revision No. 2 Effectivity date: Reviewed by: Approved by:
GEMMA E. PAGSUGIRON, MBA MA. TERESA S. SARABIA, Ph.D.
01 September 2021 QMR President

Lesson 6: List

Learning Module 6.1: List (Week 7, 6 hours)

Competence, Course Outcomes and Learning Outcomes

Competence:

Control trim, stability and stress

Course Outcome:

1. Explain the importance of maintaining stability during loading, unloading


and in-transit in various conditions.
2. Calculate the effect on trim and stability of a ship in the event of damage
to and consequent flooding of a compartment and countermeasures to be
taken.

Learning Outcomes:

At the end of the learning module, the student can:

1. Create a table of moments:


– About the keel to determine final KG and
– About the center line to determine GG H to solve for final list of multiple weight
problems e.g. ship initially upright, ship initially listed and loading weights about
the centerline to complete upright).

Discussion

6.1 Loll, or List

A ship may develop a list for one of two reasons:

a) If the center of gravity is out of the center line of the ship;


b) If the ship has a negative GM.

@ All Rights Reserved


Issue No. 1 Page 123 of 289
ST. THERESE- MTC COLLEGES SEAMANSHIP 6
Iloilo, Philippines (Advance Trim, Stability and Stress)
STUDENT LEARNING MODULE

E
Revision No. 2 Effectivity date: Reviewed by: Approved by:
GEMMA E. PAGSUGIRON, MBA MA. TERESA S. SARABIA, Ph.D.
01 September 2021 QMR President
These conditions are usually caused by faulty loading of the cargo and are generally
avoidable if the weights in the ship are properly distributed. They are not necessarily
dangerous, provided that the ship has an adequate range of stability, but are
obviously bad seamanship. In either case the ship will heel over until she is in
neutral equilibrium, that is, until B has moved out sufficiently to come vertically
under G. the first condition can occur in either stiff or tender ships and the list will
always be towards that side of the centerline to which G has moved. In second
condition the list may be to either side or may, under the influence of external
forces, change from one side to the other. It may also increase or decrease if
weights are taken away from, added to the ship.

6.2 Angle of List


Consider a ship floating upright as shown in Figure 6.1. The centers of gravity and buoyancy
are on the centerline. The resultant force acting on the ship is zero and the resultant
moment about the center of gravity is zero. b

W L
G
B

K
w
Figure 6.1

Now let a weight already on board the ship be shifted transversely such that G
moves to G1 as in Figure 6.2(a). This will produce a listing moment of W x GG 1, and
the ship will list until G1 and the center of buoyancy are in the same vertical line, as
in Figure 6.2(b).

b b

M
W

W L W1 θ L1
G G1 G
B G1
B B1 L

K
w

(b)
(a)
Figure 6.2

@ All Rights Reserved


Issue No. 1 Page 124 of 289
ST. THERESE- MTC COLLEGES SEAMANSHIP 6
Iloilo, Philippines (Advance Trim, Stability and Stress)
STUDENT LEARNING MODULE

E
Revision No. 2 Effectivity date: Reviewed by: Approved by:
GEMMA E. PAGSUGIRON, MBA MA. TERESA S. SARABIA, Ph.D.
01 September 2021 QMR President
On this position G1 will also lie vertically under M so long as the angle of list is small.
Therefore, if the final positions of the metacenter and the center of gravity are
known, the final list can be found, using trigonometry, in the triangle GG 1M, which is
right-angled at G.

The final position of the center of gravity is found by taking moments about the keel
and about the centerline.

6.3 Multiple Weight Problems – Ship Initially Upright


List problems are solved by taking moments about the keel to determine final KG and then
GM and then taking moments about the center line to determine GG H.

Remember rules.
 If a weight is loaded G will move directly towards the center of gravity of the loaded
weight.
 If a weight is discharged G will move directly away the center of gravity of the
discharged weight.

The procedure is as follows:

1. Take moments about the keel to determine the final KG:

Final KG =
∑ of moments about keel(t −m)
Final Displacement (t )
2. Calculate the final GM: GM = KM – KG

3. Take moments about the center line to calculate the final distance that G is off the
centerline, GGH:

(GG1) = GGH =
∑ of moments about centerline( t−m)
Final Displacement ( t)

¿H
4. Calculate the list: Tan θ LIST =
GM FINAL

@ All Rights Reserved


Issue No. 1 Page 125 of 289
ST. THERESE- MTC COLLEGES SEAMANSHIP 6
Iloilo, Philippines (Advance Trim, Stability and Stress)
STUDENT LEARNING MODULE

E
Revision No. 2 Effectivity date: Reviewed by: Approved by:
GEMMA E. PAGSUGIRON, MBA MA. TERESA S. SARABIA, Ph.D.
01 September 2021 QMR President

Process me
Example 1

A ship displaces 8000 tonnes, KG 7.60m and is initially upright. The following cargo is
worked:

Load: 300t at Kg 0.60, 6.1 m to port of CL;


250t at Kg 6.10, 7.6 m to starboard of CL:

Discharge: 50t from Kg 1.20 m, 4.6 m to port of CL;


500t from Kg 12.60 m, 4.6 m to starboard of CL.

Required:

Create a table and calculate the final angle of list on completion of cargo if the KM for the
final displacement is 9.36 m.

Weight discharged Weight loaded

500 t
M
4.6 m M

G GH G
50 t 7.6 m

4.6 m
W
6.1 m 250 t

300 t
K K
Portside Starboard side

Figure 6.3a Loading and discharging weights Figure 6.3b After completion ship listed 6.1° to Port

@ All Rights Reserved


Issue No. 1 Page 126 of 289
ST. THERESE- MTC COLLEGES SEAMANSHIP 6
Iloilo, Philippines (Advance Trim, Stability and Stress)
STUDENT LEARNING MODULE

E
Revision No. 2 Effectivity date: Reviewed by: Approved by:
GEMMA E. PAGSUGIRON, MBA MA. TERESA S. SARABIA, Ph.D.
01 September 2021 QMR President

a. Take the moments about the keel to determine the final KG.

Weight (t) KG (m) Moments (t-m)


(W x KG)
Initial Ship 8,000 7.60 60,800.00
Load 300 0.60 180.00
Load 250 6.10 1,525.00
Discharge - 50 1.20 - 60.00
Discharge - 150 12.60 - 6,300.00
Final 8,000 t 7.018 m 56,145.00 t-m

Total moments 56,145t−m


Final KG= = = 7.018 m
Total weights 8,000 t
b. Calculate the final GM. KM 9.360 m
GM = KM ~ KG KG ~ 7.018 m
GMFINAL 2.342 m

c. Take moments about the centerline to determine GG H.

Weight (t) Dist off CL Port Starboard


Moments (t-m) Moments (t-m)
8,000 0.00 0.0
+ 300 6.10 1830.0
+ 250 7.60 1900.0
- 50 4.60 230.0
- 500 4.60 2300.0
4,130.0 t-m 2130.0 t-m
~ 2,130.0 t-m
2,000.0 t-m

GGH =
∑ of moments about centerline( t−m) = 2,000 = 0.25 m
Final Displacement ( t) 8,000

d. Calculate the final list.


¿H 0.250
Tan θ LIST = = = 0.10675
GM FINAL 2.342
List θ = 0.10675 tan-1

List = 6° 5’ 35.7” (6’/60= 0.1°) = List = 6.1° Port

@ All Rights Reserved


Issue No. 1 Page 127 of 289
ST. THERESE- MTC COLLEGES SEAMANSHIP 6
Iloilo, Philippines (Advance Trim, Stability and Stress)
STUDENT LEARNING MODULE

E
Revision No. 2 Effectivity date: Reviewed by: Approved by:
GEMMA E. PAGSUGIRON, MBA MA. TERESA S. SARABIA, Ph.D.
01 September 2021 QMR President

6.4 Multiple Weight Problems – Ship Initially Listed


M
If a ship is initially listed G must be off the center line as shown:

Since: ᶿ
¿H
Tan θ LIST =
GM FINAL
G
Then: GGH = Tanθ LISTx GM
Where GG1 is the initial listing lever to be incorporated into GH
the moments table for the ship. Consider the following example.

Figure 6.4

Process me
Example 2

A ship has a displacement of 15,000 tonnes, KG 8.600 m, KM 9.400 m and is listed 6° to


starboard. Cargo is worked as follows:

Load: 150 t at Kg 7.60, 5.0 m to port of CL;


305 t at Kg 8.00 on the CL:
95 t at Kg 8.00 m, 4.2 m to starboard of CL;

Calculate the final angle of list. (Assume KM remains constant).

Solution:
a. Calculate the initial GM.
GM = KM ~ KG
GM = 9.400 m ~ 8.400 m= 0.800 m

b. Calculate GGH:
GGH = Tanθ LISTx GM
GGH = Tan 6° x 0.800
GGH = 0.10510 x 0.800
GGH= 0.084 m

@ All Rights Reserved


Issue No. 1 Page 128 of 289
ST. THERESE- MTC COLLEGES SEAMANSHIP 6
Iloilo, Philippines (Advance Trim, Stability and Stress)
STUDENT LEARNING MODULE

E
Revision No. 2 Effectivity date: Reviewed by: Approved by:
GEMMA E. PAGSUGIRON, MBA MA. TERESA S. SARABIA, Ph.D.
01 September 2021 QMR President

4° List
6° List to stbd
M to stbd M
W L
G GH G GH
4.2 m
305 t

150 t 95 t
5.0 m

K K

Figure 6.5a Ship listed 6° to starboard Figure 6.5b Ship listed 4° to starboard after loading

c. Take the moments about the keel to determine the final KG.

Weight (t) KG (m) Moments (t-m)


(W x KG)
Initial Ship 15,000 8.60 129,000.0
Load 150 7.60 1,140.0
Load 305 8.00 2,440.0
Load 95 8.00 760.0
Final 15,550 t 8.575 m 133,340.0 t-m

Total moments 133,340t−m


Final KG= = = 8.575 m
Total weights 15,550 t

d. Calculate the final GM.


KM 9.400 m
GMF = KM ~ KG KG ~ 8.575 m
GMFINAL 0.825 m

@ All Rights Reserved


Issue No. 1 Page 129 of 289
ST. THERESE- MTC COLLEGES SEAMANSHIP 6
Iloilo, Philippines (Advance Trim, Stability and Stress)
STUDENT LEARNING MODULE

E
Revision No. 2 Effectivity date: Reviewed by: Approved by:
GEMMA E. PAGSUGIRON, MBA MA. TERESA S. SARABIA, Ph.D.
01 September 2021 QMR President
e. Take moments about the centerline to determine GG H.

Weight (t) Dist off CL Port Starboard


Moments (t-m) Moments (t-m)
15,000 0.084 1,260.0
+ 150 5.000 750.0
+ 305 0.000 0.0 0.0
+95 4.200 399.0
750.0 t-m 1,659.0 t-m
~ 750.0
909.0

GGH =
∑ of listing moments about centerline(t−m)
Final Displacement (t )

909
GGH = = 0.058 m
15,550

f. Calculate the final list.


¿H
Tan θ LIST = Tan θ LIST =
GM FINAL

0.058
Tan θ LIST = = 0.07030
0.825

List θ = 0.07030 tan-1

List = 4° 1’ 17.22” (1’/60= 0.0°)

Final List = 4.0° Starboard

6.5 Loading Weights about the Center Line to Complete Upright

A common question arises where the ship is near completion of loading and the remaining
cargo has to be distributed between two compartments that are either side of the center
line in such a way that ship completes upright.

To complete upright: Port moments MUST EQUALT to starboard moments .

@ All Rights Reserved


Issue No. 1 Page 130 of 289
ST. THERESE- MTC COLLEGES SEAMANSHIP 6
Iloilo, Philippines (Advance Trim, Stability and Stress)
STUDENT LEARNING MODULE

E
Revision No. 2 Effectivity date: Reviewed by: Approved by:
GEMMA E. PAGSUGIRON, MBA MA. TERESA S. SARABIA, Ph.D.
01 September 2021 QMR President

Process me
Example 3

The ship initial displacement 17,500 tonnes, KG 8.800 m, KM 9.400 m and listed 3º to
starboard. 400 tonnes of cargo remains to be loaded where space is available in a tween
deck Kg 10.50 m, 7.0 m to port of CL and 10.0 m to starboard of CL. Assume KM remains
constant. Calculate the final GM and the amount of cargo to load in each space so that the
ship will complete loading upright.

Solution: METHOD 1

a. Calculate the initial GM.

Initial GM = KM ~ Initial KG
Initial GM = 9.40 m ~ 8.800 m= 0.600 m

b. Calculate GGH:
GGH = Tanθ LISTx GM
GGH = Tan 3° x 0.600 m
GGH= 0.031 m

c. Take the moments about the keel to determine the final KG and GM. (note that all 400 t
of cargo is loaded at KG 10.5 m so treat as a single weight).

Weight (t) KG (m) Moments (t-m)


Initial Ship 17,500 8.80 154,000
Load 400 10.50 4,200
Final 17,900 t 8.837 m 158,200 t-m

Total moments 158,200t−m


Final KG= = = 8.837 m
Total weights 117,900 t

d. Calculate the final GM.

GM Final = KM ~ KG KM 9.400 m
KG ~ 8.837 m
GMFINAL 0.563 m

e. Take moments about the centerline to determine GG H.

@ All Rights Reserved


Issue No. 1 Page 131 of 289
ST. THERESE- MTC COLLEGES SEAMANSHIP 6
Iloilo, Philippines (Advance Trim, Stability and Stress)
STUDENT LEARNING MODULE

E
Revision No. 2 Effectivity date: Reviewed by: Approved by:
GEMMA E. PAGSUGIRON, MBA MA. TERESA S. SARABIA, Ph.D.
01 September 2021 QMR President
Weight (t) Dist. off CL Port Starboard
Moments (t-m) Moments (t-m)
17,500 0.031 542.5
+ 400 7.000 2,800
2,800 t-m 542.5 t-m
~ 542.5
2,257.5 t-m

If all 400 tonnes loaded onto the port side space the ship would complete with an excess
of 2,257.5 t-m moments to port. Therefore some of this 400 t must now be shifted to the
space on the starboard side a distance of 17.0 meters (7.0 m + 10.0 m).

Where:

2,257.5 t-m = w x d
2,257.5 t-m = w x (7.0 m + 10.0m)
2,257.5 t-m = 17w
2,257.5t−m
w= = 132.8 tonnes to be shifted from port to starboard
17 m
To complete the upright:
Load = 400 tonnes – 132.8 tonnes = 267.2 tonnes to port
Solution: METHOD 2
a. Calculate the initial GM.

Initial GM = KM ~ Initial KG
Initial GM = 9.40 m ~ 8.800 m= 0.600 m

b. Calculate GGH:
GGH = Tanθ LISTx GM
GGH = Tan 3° x 0.600 m = 0.031 m
c. Take the moments about the keel to determine the final KG and GM. (note that all 400 t
of cargo is loaded at KG 10.5 m so treat as a single weight).

Weight (t) KG (m) Moments (t-m)


Initial Ship 17,500 8.80 154,000
Load 400 10.50 4,200
Final 17,900 t 8.837 m 158,200 t-m

Total moments 158,200t−m


Final KG= = = 8.837 m
Total weights 117,900 t

@ All Rights Reserved


Issue No. 1 Page 132 of 289
ST. THERESE- MTC COLLEGES SEAMANSHIP 6
Iloilo, Philippines (Advance Trim, Stability and Stress)
STUDENT LEARNING MODULE

E
Revision No. 2 Effectivity date: Reviewed by: Approved by:
GEMMA E. PAGSUGIRON, MBA MA. TERESA S. SARABIA, Ph.D.
01 September 2021 QMR President

d. Calculate the final GM.


KM 9.400 m
GM Final = KM ~ KG KG ~ 8.837 m
GMFINAL 0.563 m

e. Take moments about the centerline: Let x = cargo to load to port; (400 – x) = cargo to
load to starboard.

Weight (t) Dist off CL Port Starboard


Moments (t-m) Moments (t-m)
17,500 0.031 542.5
x 7.000 7x
(400-x) 10.000 (4000 – 10x)
7x 542.5 + (4000 – 10x)

To complete upright:
Port moments MUST EQUALT to starboard moments. Therefore:
7x = 542.5 + (4000 – 10x)
7x = 542.5 + 4000 – 10x
7x + 10x = 542.5 + 4000
17x = 4542.5
4542.5
x= = 267.2 tonnes to port
17
Load = 400 t – 267.2 t = 132.8 tonnes to starboard

6.5 List and Free Surface Effect


List with no free surface

List with no free surface

GV GH

G GH

CL Figure 6.4

The basic list triangle is GGHM. GM is the solid metacentric height, the GM that would exist if
the ship had no slack tanks.

@ All Rights Reserved


Issue No. 1 Page 133 of 289
ST. THERESE- MTC COLLEGES SEAMANSHIP 6
Iloilo, Philippines (Advance Trim, Stability and Stress)
STUDENT LEARNING MODULE

E
Revision No. 2 Effectivity date: Reviewed by: Approved by:
GEMMA E. PAGSUGIRON, MBA MA. TERESA S. SARABIA, Ph.D.
01 September 2021 QMR President
GGH is the distance that G is off the centerline.

GGV is the virtual rise of G due to tank free surfaces. Since GM is reduced to G VM (the Fluid
GM) it can be seen that the angle of list has increased for the same distance that G is off the
center line (GGH).

The greater the free surface moments/free surface effect, the greater will be the list for the
same listing moments.

Process me
Example 4

A ship displacement 15,200 tonnes with a KG 0f 10.6 m and is initially upright. Ballast water
RD 1.022 is pumped into a rectangular Double Bottom tank length 24 m, breadth 10 m to a
sounding of 4.10 m. If the KG of the ballast water is 2.10 m and its transverse center of
gravity (TCG) is 5.15 m to the starboard of the center line calculate the final angle of list.
Assume the KM for the final displacement is 11.75 m.

1. Assuming no free surface moments;


2. Assuming for free surface moments.

Solution:

Mass of ballast water loaded = L x B x Depth x RD SW


Mass of ballast water loaded = 24 m x 10 m x 4 m x 1.025 = 984 tonnes

1. No Free Surface Moments


Weight (t) KG (m) Moments (t-m)
Initial Displacement 15,200 10.6 161,120
Load + 984 2.10 2,066.4
Final 16,184 10.08 163,186.4

KG = 10.08 m
Take the moments about the keel to determine final KG and GM.
GM Final = KM ~ KG
GM Final = 11.75 ~ 10.08
GM Final = 1.67 m

Take the moments about the center line to calculate the GG H.

Weight (t) Dist off CL Port Starboard


Moments (t-m) Moments (t-m)

@ All Rights Reserved


Issue No. 1 Page 134 of 289
ST. THERESE- MTC COLLEGES SEAMANSHIP 6
Iloilo, Philippines (Advance Trim, Stability and Stress)
STUDENT LEARNING MODULE

E
Revision No. 2 Effectivity date: Reviewed by: Approved by:
GEMMA E. PAGSUGIRON, MBA MA. TERESA S. SARABIA, Ph.D.
01 September 2021 QMR President
15,200 0.00 0.0 0.0
+ 984 5.15 5,067.6
0.0 5,067.6

Net Listing moments 5,067.6t −m


GGH = = = 0.313 m
Final displacement 16,184 t
¿H
Calculate the final list: Tan θ LIST =
GM FINAL
0.313 m
Tan θ LIST =
1.67 m
θ LIST = 0.18743 tan-1
θ LIST = 10º 36’ 56.81”
θ LIST = 10.6º to Starboard
2. Including Free Surface Moments:
lb 3
FSMs = x dt
12

24 x 103 24 x 1000 x 1.025 24,600


FSMs = x 1.025 = = = 2,050 t-m
12 12 12

Take the moments about the keel to determine the final KG and GM:
Weight (t) KG (m) Moments (t-m)
Initial Displacement 15,200 10.6 161,120
Load + 984 2.10 2,066.4
FSMs 2,050.0
Final 16,184 10.21 165,236.4
GM Final = KM ~ KG
GM Final = 11.75 m ~ 10.21 m
GM Final = 1.54 m
The value of GGH is not affected by free surface moments.
¿H
Calculate the final list: Tan θ LIST =
GM FINAL
0.313 m
Tan θ LIST =
1.54 m
θ LIST = 0.20325 tan-1
θ LIST = 11º 29’ 17.95”
θ LIST = 11.4º to Starboard

Do This

Activity 1 – List Calculation Multiple Weights


@ All Rights Reserved
Issue No. 1 Page 135 of 289
ST. THERESE- MTC COLLEGES SEAMANSHIP 6
Iloilo, Philippines (Advance Trim, Stability and Stress)
STUDENT LEARNING MODULE

E
Revision No. 2 Effectivity date: Reviewed by: Approved by:
GEMMA E. PAGSUGIRON, MBA MA. TERESA S. SARABIA, Ph.D.
01 September 2021 QMR President
A ship has a displacement of 8,820 t, KG 6.73 m and is upright. She then loads:
540 tonnes at 3.00 meters above the keel and on the centerline;
300 tonnes at 9.00 meters above keel and 2.5 m to port of centerline
200 tonnes at 5.00 meters above keel and 6.8 m to port of centerline
280 tonnes at 7.50 meters above keel and 4.5 m to starboard of centerline
If KM is then 7.81 m. Calculate the angle of heel.

Activity 2 – Loading Weights about the Center Line to Complete Upright


A ship displaces 6,400 tonnes, has a KG of 5.97 m, KM 7.62 m and is listed 5º to starboard,
500 tonnes of cargo are to be loaded into the wings of a tween deck at a distances of 5.00
off the center line to port and 7.00 m off the center line to starboard. Calculate the weight
to load in each wing in order that the ship may then be upright. (You can use any method).

Assessment 1 – List Calculation Multiple Weights

A ship of 9,500 tonnes displacement is listed 3.5º to starboard and has KM of 9.50 m and
KG 9.30 m. She loads 300 tonnes of bunkers in No. 3 DB tank portside (KG 0.60 m and
center of gravity 6.00 m from the centerline) and discharges two parcels of cargo each of 50
tonnes from the port side of No.2 shelter deck (KG 11 m and center of gravity 5.00 m from
the centerline). Calculate the final list.

Assessment No. 2– Loading Weights About the Center


of Line to Complete Upright

A ship of 12,500 tonnes displacement KM of 7.00 m and KG 6.40 m has a list 3º list to
starboard and has yet to load 500 tonnes of cargo. There is space available in the tween
decks, centers of gravity 6.00 m each side of the centerline. Find how much cargo to load
on each side if the ship is to complete loading upright.
Lesson 7: Trim

Learning Module 7.1: Trim (Week 8, 6 hours)

@ All Rights Reserved


Issue No. 1 Page 136 of 289
ST. THERESE- MTC COLLEGES SEAMANSHIP 6
Iloilo, Philippines (Advance Trim, Stability and Stress)
STUDENT LEARNING MODULE

E
Revision No. 2 Effectivity date: Reviewed by: Approved by:
GEMMA E. PAGSUGIRON, MBA MA. TERESA S. SARABIA, Ph.D.
01 September 2021 QMR President
Competence, Course Outcomes and Learning Outcomes

Competence:

Control trim, stability and stress

Course Outcome:

1. Explain the importance of maintaining stability during loading, unloading


and in-transit in various conditions.
2. Calculate the effect on trim and stability of a ship in the event of damage
to and consequent flooding of a compartment and countermeasures to be
taken.

Learning Outcomes:

At the end of the learning module, the student can:

1. Calculate the following:


– Trim;
– Change of Trim;
– Moment to change trim by one centimeter (MCTC);
– Change of Draft aft and forward due to change of trim;
– Bodily sinkage or rise ;
– Displacement and weight added or removed.

Discussion

7.1 Trim
This is the longitudinal equivalent of heel, but whereas the latter is measured in angle, trim
is measured by the difference of drafts fore and aft as measured at the forward and aft
perpendiculars.

If the drafts are the same, fore and aft, or difference is zero then the ship said to be “on
even keel”.

@ All Rights Reserved


Issue No. 1 Page 137 of 289
ST. THERESE- MTC COLLEGES SEAMANSHIP 6
Iloilo, Philippines (Advance Trim, Stability and Stress)
STUDENT LEARNING MODULE

E
Revision No. 2 Effectivity date: Reviewed by: Approved by:
GEMMA E. PAGSUGIRON, MBA MA. TERESA S. SARABIA, Ph.D.
01 September 2021 QMR President

Aft draft 5.00 m Forward draft 5.00 m

Figure 7.1 Even Keel

If the forward draft is greater that aft draft, the vessel is trimmed by the bow.

Aft draft 3.00 m Forward draft 5.50 m

Forward Trim

Figure 7.2 2.50 m Trimmed by the bow.

If the aft draft is greater than the forward draft, the vessel is trimmed by the stern.

Aft draft Forward draft 3.00 m


8.00 m
Aft Trim
Figure 7.3 5.00 m Trimmed by the stern

Causes of Trim:
1. Moving loads/mass fore and aft from one point to another changes the trim of the
ship but not the mean draft
2. Adding or removing loads to and from the ship will involve a decrease or increase of
the mean draft as well as a change in trim.
3. The change in water density where the ship floating changes the mean draft as well
as the trim of the ship.

7.2 Change of Trim


If a weight is shifted longitudinally the ship will experience a change of trim.

Process me
@ All Rights Reserved
Issue No. 1 Page 138 of 289
ST. THERESE- MTC COLLEGES SEAMANSHIP 6
Iloilo, Philippines (Advance Trim, Stability and Stress)
STUDENT LEARNING MODULE

E
Revision No. 2 Effectivity date: Reviewed by: Approved by:
GEMMA E. PAGSUGIRON, MBA MA. TERESA S. SARABIA, Ph.D.
01 September 2021 QMR President
Example 1:

A ship is floating at a drafts forward 6.100 m and aft 6.300 m. A weight is then shifted
forward. The final drafts are forward 6.090 m and aft 6.310 m. Calculate the change of trim.

6.300 m 6.100 m

Figure 7.4a: Initial Trim 0.200 m by the stern


Solution:

a. Calculate the initial trim:

Formula: Initial Trim = Forward draft ~ Aft draft

Initial Trim = 6.100 m ~ 6.300 m


Initial Trim = 0.200 m by the stern

b. Calculate the Final trim:

Formula: Final Trim = Forward draft ~ Aft draft

Final Trim = 6.090 m ~ 6.310 m


Final Trim = 0.220 m by the stern

W1

6.310 m 6.090 m

Figure 7.4b: Final trim 0.220 meters by the stern

c. Calculate the change of trim (COT):

Initial Trim = 0.200 m by the stern


Final Trim = ~ 0.220 m by the stern
Final Trim = 0.020 m by the STERN
Note: trim by the stern has increased by 0.020 m.
Example 2:

@ All Rights Reserved


Issue No. 1 Page 139 of 289
ST. THERESE- MTC COLLEGES SEAMANSHIP 6
Iloilo, Philippines (Advance Trim, Stability and Stress)
STUDENT LEARNING MODULE

E
Revision No. 2 Effectivity date: Reviewed by: Approved by:
GEMMA E. PAGSUGIRON, MBA MA. TERESA S. SARABIA, Ph.D.
01 September 2021 QMR President
A ship is floating at a draughts forward 6.000 m and aft 5.640 m. A weight is then shifted
aft. The final drafts are forward 5.680 m and aft 5.960 meters. Calculate the change of trim.

5.640 m 6.000 m

Figure 7.5a: Initial trim 0.360 meters by the head

Solution:

a. Calculate the initial trim:

Formula: Initial Trim = Forward draft ~ Aft draft


Initial Trim = 6.000 m ~ 5.640 m
Initial Trim= 0.360 m by the head

b. Calculate the initial trim:

Formula: Final Trim = Forward draft ~ Aft draft


Final Trim = 5.680 m ~ 5.960 m
Final Trim = 0.280 m by the stern

W1

5.960 m 5.680 m

Figure 7.5b: Final trim 0.280 meters by the stern

@ All Rights Reserved


Issue No. 1 Page 140 of 289
ST. THERESE- MTC COLLEGES SEAMANSHIP 6
Iloilo, Philippines (Advance Trim, Stability and Stress)
STUDENT LEARNING MODULE

E
Revision No. 2 Effectivity date: Reviewed by: Approved by:
GEMMA E. PAGSUGIRON, MBA MA. TERESA S. SARABIA, Ph.D.
01 September 2021 QMR President
c. Calculate the change of trim (COT):

Initial Trim = 0.360 m by the head


Final Trim = + 0.280 m by the stern

Final Trim = 0.640 m by the STERN

Note: The ship initially trimmed by the head, completes trimmed by the stern.

7.3 Moment to Change Trim by One Centimeter (MCT1C)


This is the trimming moment required to change the ships trim by exactly one centimeter.

It is tabulated in the ships hydrostatic particulars and is used to determine the change of
trim that takes place when weights are shifted, loaded or discharged.
Change of trim can be calculated by the formula:

Trmming moment
COT (cms) =
MCT 1 C

where the trimming moment is: w x d

“w” being the weight shifted and


“d” being the distance through which the weight is shifted longitudinally

w xd
Thus: COT (cms) =
MCT 1C

Process me
Example 1

A weight of 170 tonnes is moved aft by distance of 25 m. If the MCTC for the current
draught is 250 t-m determine the final trim of the ship if the initial trim was 0.20 m by the
stern.

25 m W
W1

Figure 7.6

@ All Rights Reserved


Issue No. 1 Page 141 of 289
ST. THERESE- MTC COLLEGES SEAMANSHIP 6
Iloilo, Philippines (Advance Trim, Stability and Stress)
STUDENT LEARNING MODULE

E
Revision No. 2 Effectivity date: Reviewed by: Approved by:
GEMMA E. PAGSUGIRON, MBA MA. TERESA S. SARABIA, Ph.D.
01 September 2021 QMR President

Solution:
a. Calculate the change of trim (COT):
w xd
COT (cms) =
MCT 1C
170t x 25 m
COT (cms) =
250 t−m
4250
COT (cms) =
250
COT (cms) = 17 cms – convert to meter (17/100)

COT = 0.17 m by the stern

b. Calculate the final trim:

Initial Trim= 0.200 m by the stern


COT = + 0.170 m further by the stern

Final Trim = 0.370 m by the STERN

7.4 Formula for Calculating MCT1C


In practice the MCT1C value will always be found for the draught in question in the ship’s
hydrostatic particulars. However, in examinations it may have to be calculated and the
formula for calculating MCT1C is:

W x GML
MCT1C =
100 LBP
Where: “W” is the displacement
“GML” is the longitudinal metacentric height
“LBP” is the length between perpendiculars

@ All Rights Reserved


Issue No. 1 Page 142 of 289
ST. THERESE- MTC COLLEGES SEAMANSHIP 6
Iloilo, Philippines (Advance Trim, Stability and Stress)
STUDENT LEARNING MODULE

E
Revision No. 2 Effectivity date: Reviewed by: Approved by:
GEMMA E. PAGSUGIRON, MBA MA. TERESA S. SARABIA, Ph.D.
01 September 2021 QMR President
The derivation of this formula is as follows:

The ship in Figure 7.7 is on even keel with a weight on deck.

W G L
B

Figure 7.7
The weight is shifted aft along the deck through distance “d” meters.in accordance with the
formula:
wxd
GG1 =
W
G and B become horizontally separated creating a trimming lever. This causes the ship to
trim by the stern until B attains a new position vertically below the new longitudinal center
of gravity, G1 (Figure 7.9).

W1 W

G1 G

Figure 7.8

@ All Rights Reserved


Issue No. 1 Page 143 of 289
ST. THERESE- MTC COLLEGES SEAMANSHIP 6
Iloilo, Philippines (Advance Trim, Stability and Stress)
STUDENT LEARNING MODULE

E
Revision No. 2 Effectivity date: Reviewed by: Approved by:
GEMMA E. PAGSUGIRON, MBA MA. TERESA S. SARABIA, Ph.D.
01 September 2021 QMR President
GG1 ML is a right angled triangle where:

OPP
Tan θ =
ADJ

¿1
Therefore: Tan θ =
GML

wxd ¿1
Since: GG1 = andTan θ = then:
W GML

wxd
Tan θ =
W x GML

TRIM ( m)
Also in Figure 7.10: Tan θ =
LBP(m)

ML
θ

W
W1
L
W1 G1 G
L1
B1 B
W

Figure 7.9

AP FP

W1

TRIM
LBP

Figure 7.10
If the change of trim due to the weight shifted is exactly 1 cm, then:

TRIM ( m) 0.01(m) 1
Tan θ =
LBP(m)
=Tan θ =
LBP( m)
= Tan θ =
100 LBP
(1)

@ All Rights Reserved


Issue No. 1 Page 144 of 289
ST. THERESE- MTC COLLEGES SEAMANSHIP 6
Iloilo, Philippines (Advance Trim, Stability and Stress)
STUDENT LEARNING MODULE

E
Revision No. 2 Effectivity date: Reviewed by: Approved by:
GEMMA E. PAGSUGIRON, MBA MA. TERESA S. SARABIA, Ph.D.
01 September 2021 QMR President

wxd MCTC
Since: Tan θ = equals: Tan θ = (2)
W x GML W x GML

(Because (w x d) is the moment to change the trim by exactly 1 cm)

Bringing formulae (1) and (2) together gives:

1 MCTC
=
100 LBP W x GML

W x GML
Rearranging this gives: MCT1C =
100 LBP

Process me
Example

A ship is 125 meters long. Has a displacement of 3,650 tonnes and a longitudinal
metacentric height of 200 meters. Calculate her Moment to Change Trim by 1 Centimeter
(MCT1C).
W x GML 3,650t x 200 m 730,000
MCT1C = = = = 58.4 tonne-meters
100 LBP 100 x 125 m 12,500

7.5 Longitudinal Center of Flotation (LCF or F)


This is at the geometric center of the ship’s water plane area and is the point about which
the ship will trim.

Consider the ship where a weight is shifted longitudinally.

W1 W

W1
L
F
W
L1
Figure 7.11 (a)

@ All Rights Reserved


Issue No. 1 Page 145 of 289
ST. THERESE- MTC COLLEGES SEAMANSHIP 6
Iloilo, Philippines (Advance Trim, Stability and Stress)
STUDENT LEARNING MODULE

E
Revision No. 2 Effectivity date: Reviewed by: Approved by:
GEMMA E. PAGSUGIRON, MBA MA. TERESA S. SARABIA, Ph.D.
01 September 2021 QMR President
It can be seen that the LCF is in the same position in the ship’s length as the point where
the initial and final waterlines intersect.

Because the water plane area changes shape and size with draught the position of the LCF
will also change with draught.

The position of the LCF is normally quoted in hydrostatic data as being so many meters
forward of the after perpendicular (foap).

The position of the LCF is important because if a ship experiences a change of trim, some of
that change of trim must be applied to the aft draught and the remainder applied to the
forward draught as can be seen.

W1

L
W1
F1 L1
W

CL
Figure 7.11 (b)

7.6 Change of Draft Forward and Aft Due to Change of Trim

When a ship changes trim it will obviously cause a change in the drafts forward and aft. One
of these will be increased and the other decreased. A formula must now be found that will
give the change in drafts due to change in trim.

Consider a ship floating upright, as shown in Figure 7.12 (a). F1represents the position of
the center of flotation, which is l meters from aft. The ship’s length is L meters and a weight
“w” is on deck forward. Let this weight now be shifted aft a distance of “d” meters. The ship
will trim about F1 and change the trim “t” cm by the stern, as shown in Figure 7.11 (b).
W1C is a line drawn parallel to the keel. “A” represents the new draft aft and “F” the new
draft forward. The trim is therefore equal to A – F and, since the original trim was zero, this
must also be equal to the change of trim.

@ All Rights Reserved


Issue No. 1 Page 146 of 289
ST. THERESE- MTC COLLEGES SEAMANSHIP 6
Iloilo, Philippines (Advance Trim, Stability and Stress)
STUDENT LEARNING MODULE

E
Revision No. 2 Effectivity date: Reviewed by: Approved by:
GEMMA E. PAGSUGIRON, MBA MA. TERESA S. SARABIA, Ph.D.
01 September 2021 QMR President

L = LBP

W F1 L

AP FP
w
Figure 7.12 (a)

W1 W
C
W1 F1 L t
L1
A W

CL F
X l
Figure 7.12 (b)

Let “x” represent the change of draft aft due to the change of trim and let “y” represent the
change forward. In the triangles WW1F1 and W1L1C, using the property of similar triangles:

x cm t cm lm x t cm
= or x cm =
1m L m Lm
l
Therefore Change of draft aft in cm = x Change of trim in cm
L

where l = the distance of center of flotation from aft in meters and


L = the ships length in meters
It will also be noticed that x + y = t.

Therefore: Change of draft Forward in cm = Change of trim – Change of draft Aft

@ All Rights Reserved


Issue No. 1 Page 147 of 289
ST. THERESE- MTC COLLEGES SEAMANSHIP 6
Iloilo, Philippines (Advance Trim, Stability and Stress)
STUDENT LEARNING MODULE

E
Revision No. 2 Effectivity date: Reviewed by: Approved by:
GEMMA E. PAGSUGIRON, MBA MA. TERESA S. SARABIA, Ph.D.
01 September 2021 QMR President

Process me

Example 1: The Effect of Shifting a Weight

A ship is 126 m long is floating at draft of 6.50 m forward and 7.50 m aft. The center of
flotation is 3 m aft of amidships. MCT1C is 250 tonnes-meter. Displacement is 6000 tonnes.
Calculate the new drafts if a weight of 120 tonnes already on board is shifted forward a
distance of 50 meters.

L =126 m

d=45m
W W1

W l = 60 m l = 63 + 3 m = 66m L
F 3m

AP FP
Amidship
Figure 7.13
a. Trimming moment = w x d
= 120 t x 50 m
Trimming moment = 6000 t-m by the head

Trimming moment 6000t−m


b. Change of trim (CT) = =
MTC 1 C 250t−m

Change of trim (CT) = 24 cm by the head


l
c. Change of draft aft = x Change of trim (CT)
L

L 126 m
Where: l= 2 = 2
= 63m – 3m = 60 m
60 m
Change of draft aft = x 24 cm
126 m

@ All Rights Reserved


Issue No. 1 Page 148 of 289
ST. THERESE- MTC COLLEGES SEAMANSHIP 6
Iloilo, Philippines (Advance Trim, Stability and Stress)
STUDENT LEARNING MODULE

E
Revision No. 2 Effectivity date: Reviewed by: Approved by:
GEMMA E. PAGSUGIRON, MBA MA. TERESA S. SARABIA, Ph.D.
01 September 2021 QMR President
60 x 24 cm
Change of draft aft =
126
Change of draft aft = 11.43 cm (convert to meters)
Change of draft aft = 0.11 m

l
d. Change of draft forward = x Change of trim (CT)
L
Where: l = 63m + 3m = 66 m
66 m
Change of draft forward = x 24 cm
126 m
66 x 24 cm
Change of draft forward =
126
Change of draft forward = 12.57 cm (convert to meters)

Change of draft forward = 0.13 m

e. Calculate the Final draughts:


Aft Forward
Initial draughts 7.50 m 6.50 m
Change of trim _- 0.11 m + 0.13 m
New Draughts 7.39 m 6.63 m

Example 2

Calculate the new drafts in a ship of 125 m length, when a weight of 100 tonnes is shifted
aft for a distance of 60 m from the tipping center. The initial drafts were 4.80 m forward
and 4.76 m aft. The center of flotation is 2.5 m abaft amidships and the MCT1C is 120 t-m.

L =125 m

d=60m

W1 W

W l= 62.5 – 2.5 m = 60m L


F 2.5 m l = 62.5 + 2.5 = 65 m

AP FP
Amidship
Figure 7.14

@ All Rights Reserved


Issue No. 1 Page 149 of 289
ST. THERESE- MTC COLLEGES SEAMANSHIP 6
Iloilo, Philippines (Advance Trim, Stability and Stress)
STUDENT LEARNING MODULE

E
Revision No. 2 Effectivity date: Reviewed by: Approved by:
GEMMA E. PAGSUGIRON, MBA MA. TERESA S. SARABIA, Ph.D.
01 September 2021 QMR President
wxd 100t x 60 m
a. Change of trim (CT) =
MTC 1C
=
120 t−m
= 50 cm by the stern

LCF (F) is 65 m from forward and 60 m from aft


l
b. Change of draft aft = x Change of trim (CT)
L

L 125 m
Where: l= 2= 2 = 62.5m – 2.5m = 60 m
60 m
Change of draft aft = x 50 cm
125 m
60 x 50 cm
Change of draft aft =
125
Change of draft aft = 24 cm (convert to meters)
Change of draft aft = 0.24 m

c. Change of draft forward = Change of trim (CT) – Change of draft aft

Change of draft forward = 0.50m – 0.24m


Change of draft forward = 0.26 m

d. Calculate the Final draughts:


Aft Forward
Initial draughts 4.76 m 4.80 m
Change of trim _+0.24 m - 0.26 m
New Draughts 5.00 m 4.54 m

Example 3

A box-shaped vessel 100m x 15m x 6 m depth floats in salt water on an even keel at 4.00 m
draft (see Figure 7.15). Calculate the final drafts if a weight of 65 tonnes already on board is
shifted a distance of 45 meters aft.
64t
40 m 64t
L
W LCF L W1 LCF L1

W l F
A F
A
Figure 7.15
Salt water density: 1.025
a. Calculate for BML:
2
L
Formula: BML =
12d

@ All Rights Reserved


Issue No. 1 Page 150 of 289
ST. THERESE- MTC COLLEGES SEAMANSHIP 6
Iloilo, Philippines (Advance Trim, Stability and Stress)
STUDENT LEARNING MODULE

E
Revision No. 2 Effectivity date: Reviewed by: Approved by:
GEMMA E. PAGSUGIRON, MBA MA. TERESA S. SARABIA, Ph.D.
01 September 2021 QMR President
100 x 100 10,000
BML = =
12 x 4 48

BML = 208.3 m

b. Calculate for W:
W = L X B x d x dt
W = 100 x 15 x 4 x 1.025
W = 6,150 tonnes

c. Calculate for MCT1C:


Since BG is small compared with GML. BML can be used instead of GML:

W x BML 6,150t x 208.3 m


MCT1C = =
100 LBP 100 x 100 m
1,281,045
MCT1C = =128.1 t-m/cm
10,000

d. Calculate for Change of Trim:


wxd
Change of trim (CT) =
MTC 1C
65 t x 45 m
=
128.1t−m
2,925 m
=
128.1t−m/cm

Change of trim (CT) = 22.83 cm by the stern


Change of trim (CT) = 23 cm by the stern

e. Calculate for Change of drafts:


l
Change of draft aft = x Change of trim (CT)
L

1
Change of draft aft = x 23 cm
2
23 cm
Change of draft aft =
2
Change of draft aft = 11.5 cm (convert to meters)
Change of draft forward = 0.12 m

f. Calculate the Final draughts:


Aft Forward
@ All Rights Reserved
Issue No. 1 Page 151 of 289
ST. THERESE- MTC COLLEGES SEAMANSHIP 6
Iloilo, Philippines (Advance Trim, Stability and Stress)
STUDENT LEARNING MODULE

E
Revision No. 2 Effectivity date: Reviewed by: Approved by:
GEMMA E. PAGSUGIRON, MBA MA. TERESA S. SARABIA, Ph.D.
01 September 2021 QMR President
Initial draughts 4.00 m 4.00 m
Change of trim _+ 0.12 m - 0.12 m
New Draughts 4.115 m 3.885 m

7.7 Trim Calculations – Changing of Loading


7.7.1 The effect of loading, Discharging and Moving Weights
When a weight is loaded at the center of flotation it will produce no trimming moment, but
the ship’s drafts will increase uniformly so that the ship displaces an extra weight of water
equal to the weight loaded. If the weight is now shifted forward or aft away from the center
of flotation, it will cause a change of trim. From this it can be seen that when a weight is
loaded away from the center of flotation, it will cause both a bodily sinkage and a change of
trim.

Similarly, when a weight is being discharged, if the weight is first shifted to the center of
flotation it will produce a change of trim, and if it is then discharged from the center of
flotation the ship will rise bodily. Thus, both a change of trim and bodily rise must be
considered when a weight is being discharged away from the center of flotation.

If weights are loaded or discharged the effect to bodily rise or sinkage must also be
considered.

The following procedure should be followed when loading a weight.


1. First, assume the weight to be added at the center of flotation and calculate the
sinkage by the formula:
w
Bodily Sinkage/Rise cms=
TPC

2. Next, assume the weight to be shifted from the center of flotation, forward or aft, to
its new position. Calculate the change of trim caused by this by the formula:
wxd
Chang of trim (CT) =
MTC 1C
3. Calculate the change of draft at either end, due to the above change of trim, as
described in “Change of draft due to change of trim”.
4. Add the results of (a) to the draft at each end: then apply the results of (c). This will
give the ship’s new drafts after the weight has been added.
5. When weights are discharged, we follow the same procedures as for weights loaded.
Remember, however, that they will cause a bodily rise instead of sinkage: also be
careful about which way will cause the trim to change.

@ All Rights Reserved


Issue No. 1 Page 152 of 289
ST. THERESE- MTC COLLEGES SEAMANSHIP 6
Iloilo, Philippines (Advance Trim, Stability and Stress)
STUDENT LEARNING MODULE

E
Revision No. 2 Effectivity date: Reviewed by: Approved by:
GEMMA E. PAGSUGIRON, MBA MA. TERESA S. SARABIA, Ph.D.
01 September 2021 QMR President
6. If a number of weights are loaded or discharged, use the total effects of all of them.
The bodily rise or sinkage will equal the net weight, divided by the TPC. The moment
changing trim will be the algebraic sum of all the moments.

Process me
Example 1

A ship 90 m long is floating at draft of 5.50 m forward and 6.00 m aft. The center of
flotation is 1.5 m aft of amidships, TPC = 12 tonnes and MCT1C is 120 t-m (see figure
7.15). Calculate the new drafts if a total weight of 450 tonnes is loaded in a position 14 m
forward of amidships.

L =90 m

l= 45 m
450 t

d=1.5+14=15.5m
W L
14m
la= 43.5 m F 1.5 m

AP FP
D Aft = 6.00 m Figure 7.16 D Fwd = 5.50
m
a. Calculate bodily sinkage:
w
Bodily Sinkage =
TPC
450 t
Bodily Sinkage =
12t /cm
Bodily Sinkage = 37.5 cm (convert to meters)

Bodily Sinkage= 0.375 m

b. Calculate Change of Trim:

d= 1.5 m + 14 m = 15.5 m
wxd
Chang of trim (CT) =
MTC 1C

@ All Rights Reserved


Issue No. 1 Page 153 of 289
ST. THERESE- MTC COLLEGES SEAMANSHIP 6
Iloilo, Philippines (Advance Trim, Stability and Stress)
STUDENT LEARNING MODULE

E
Revision No. 2 Effectivity date: Reviewed by: Approved by:
GEMMA E. PAGSUGIRON, MBA MA. TERESA S. SARABIA, Ph.D.
01 September 2021 QMR President
450 t x 15.5 m 6,975t−m
Change of trim (CT) = =
120 t−m/cm 120t−m/cm
Change of trim (CT) = 58.13 cm by the head

c. Calculate Change of drafts:


l
Change of draft aft = x Change of trim (CT)
L
where: l = 43.5 m (from aft to LCF)
43.5 m
Change of draft aft = x 58.13 cm
90 m
2,528.66 cm
Change of draft aft =
90
Change of draft aft = 28.10 cm (convert to meters)

Change of draft aft =0.281 m

d. Change of draft forward =Change of trim (CT) – Change of draft aft

Change of draft forward = 0.581m –0.281m


Change of draft forward = 0.300 m
e. Calculate the Final draughts:
Aft Forward
Initial draughts 6.000 m 5.500 m
Bodily sinkage + 0.375 m + 0.375 m
6.375 m 5.875 m
Change due trim - 0.281 m + 0.300 m
New Draughts = 6.094 m 6.175 m

@ All Rights Reserved


Issue No. 1 Page 154 of 289
ST. THERESE- MTC COLLEGES SEAMANSHIP 6
Iloilo, Philippines (Advance Trim, Stability and Stress)
STUDENT LEARNING MODULE

E
Revision No. 2 Effectivity date: Reviewed by: Approved by:
GEMMA E. PAGSUGIRON, MBA MA. TERESA S. SARABIA, Ph.D.
01 September 2021 QMR President
Example 2

A box-shaped vessel 40 m x 6 m x 3 m depth floats in salt water on an even keel at 3.00 m


draft (see Figure 7.17). Calculate the final drafts if a weight of 45 tonnes is discharged from
a position 6 m from forward. MCT1C IS 8.4 t-m.

F1 L

F
A
Figure 7.17
a. Calculate for BML:
WPA
Formula: TPC=
97.56
40 m X 6 m 240 m
TPC= =
97.56 97.56

TPC = 2.46 tonnes/cm

b. Calculate bodily rise:


w
Bodily rise=
TPC
45 t
Bodily rise= = 18.29 cm = 0.183 m
2.46 t/cm
c. Calculate for Change of Trim:
wxd
Change of trim (CT) =
MTC 1C
45 t x 14 m
=
8.4 t−m
630 m
=
8.4 t −m/cm
Change of trim (CT) = 75 cm by the stern

d. Calculate Change of drafts:


l
Change of draft aft = x Change of trim (CT)
L
1 75 cm
Change of draft aft = x 75 cm =
2 2

Change of draft aft = 37.5 cm (convert to meters)

@ All Rights Reserved


Issue No. 1 Page 155 of 289
ST. THERESE- MTC COLLEGES SEAMANSHIP 6
Iloilo, Philippines (Advance Trim, Stability and Stress)
STUDENT LEARNING MODULE

E
Revision No. 2 Effectivity date: Reviewed by: Approved by:
GEMMA E. PAGSUGIRON, MBA MA. TERESA S. SARABIA, Ph.D.
01 September 2021 QMR President
Change of draft aft = 0.375 m
l
Change of draft forward = x Change of trim (CT)
L
1 75 cm
= x 75 cm=
2 2
Change of draft forward= 37.5 cm (convert to meters)
Change of draft forward = 0.375 m

e. Calculate the Final draughts:


Aft Forward
Initial draughts 3.000 m 3.000 m
Bodily rise - 0.183 m - 0.183 m
2.817 m 2.817 m
Change due trim + 0.375 m - 0.375 m
New Draughts 3.192 m 2.442 m

Example 3

A ship 100 m long is floating at draft of 7.00 m forward and 6.80 m aft. The longitudinal
center of flotation is 45 m forward of aft perpendicular, TPC is15 tonnes and MCT1C is 150
t-m (see figure 7.18). Calculate the new drafts if a total weight of 200 tonnes is loaded 20 m
forward of aft perpendicular.

L =100 m

200t

d?
W 20m L
Aft l= 45 m F Fwd
7.00m
6.80m
AP FP
Figure 7.18

a. Calculate bodily sinkage:


w
Bodily Sinkage =
TPC

@ All Rights Reserved


Issue No. 1 Page 156 of 289
ST. THERESE- MTC COLLEGES SEAMANSHIP 6
Iloilo, Philippines (Advance Trim, Stability and Stress)
STUDENT LEARNING MODULE

E
Revision No. 2 Effectivity date: Reviewed by: Approved by:
GEMMA E. PAGSUGIRON, MBA MA. TERESA S. SARABIA, Ph.D.
01 September 2021 QMR President
200t
Bodily Sinkage =
15t / cm
Bodily Sinkage = 13.33 cm (convert to meters)

Bodily Sinkage = 0.133m

b. Calculate Change of Trim:


d=(l – distance from AP) 45 m -20 m = 25 m
wxd
Change of trim (CT) =
MTC 1C
200 t x 25 m
Change of trim (CT) =
150t−m/cm
5000t−m
=
150t−m/cm
Change of trim (CT) = 33.3 cm by the stern

c. Calculate Change of drafts aft and forward:


l
Change of draft aft = x Change of trim (CT)
L
where: l = 45 m (from aft perpendicular to LCF )
45 m
Change of draft aft = x 33.3 cm
100 m
1,498.5 cm
Change of draft aft =
100
Change of draft aft = 14.99 cm (convert to meters)
Change of draft aft = 0.150 m

Change of draft forward =Change of trim (CT) – Change of draft aft


Change of draft forward = 0.333 m –0.150 m
Change of draft forward = 0.183 m

Note: Weight is loaded aft of the LCF (F) so the ship will trim by the STERN.

d. Calculate the Final draughts:


Aft Forward
Initial draughts 6.800 m 7.000 m

@ All Rights Reserved


Issue No. 1 Page 157 of 289
ST. THERESE- MTC COLLEGES SEAMANSHIP 6
Iloilo, Philippines (Advance Trim, Stability and Stress)
STUDENT LEARNING MODULE

E
Revision No. 2 Effectivity date: Reviewed by: Approved by:
GEMMA E. PAGSUGIRON, MBA MA. TERESA S. SARABIA, Ph.D.
01 September 2021 QMR President
Bodily sinkage + 0.133 m + 0.133 m
6.933 m 7.133 m
Change due trim + 0.150 m - 0.183 m
New Draughts 7.083 m 6.950 m

7.7.2 Multiple Weights Problems

Problems involving multiple weights require a tabular approach to be adopted where


moments are taken about the LCF.

Process me
Example 1

A ship 120 m in length floats at draughts 6.24m forward and 6.36 m aft. LCF is 54 m
forward of AP, TPC 14.2 and MCT1C 116 t-m.
Note: Longitudinal center of gravity (lcg) Forward Aft Perpendicular (foap)

The following cargo is worked:


Load 120 t longitudinal center of gravity (lcg) 10.0 foap
Load 68 t longitudinal center of gravity lcg 86 foap
Discharge 36 t longitudinal center of gravity lcg 22 foap
Discharge 48 t longitudinal center of gravity lcg 60 foap
Calculate the final draughts.

Procedures:

1. The weights loaded and discharged are summed to find the net weight loaded or
discharged to be entered in column 1 of the table.
2. The weights are listed as positive values, regardless of whether the weight is being
loaded or discharged to be entered in column 2 of the table.
3. The distance that each weight is loaded or discharged from the LCF is listed in
column 3 of the table.
4. Trimming moments are calculated (Column 2 being multiplied by column 3) and
entered in column 4 or 5 depending on whether they are head or stern moments.(It
is here that mistakes are commonly made whereby the moments are often applied
the wrong way).

@ All Rights Reserved


Issue No. 1 Page 158 of 289
ST. THERESE- MTC COLLEGES SEAMANSHIP 6
Iloilo, Philippines (Advance Trim, Stability and Stress)
STUDENT LEARNING MODULE

E
Revision No. 2 Effectivity date: Reviewed by: Approved by:
GEMMA E. PAGSUGIRON, MBA MA. TERESA S. SARABIA, Ph.D.
01 September 2021 QMR President
Solution:

120m

60m 48 t
22m 36t d=32m d=6m
54m .F 86m

10m d=44m d=32m


120 68 t

l = 60m FP
AP

Figure 7.19

a. Calculate the distances each cargo from F:

Longitudinal Center of Flotation (LCF or F) = 54 m foap.


Distance of Load 120 t (54m – 10 m) = 44 m
Distance of Load 68 t (86 m – 54 m) = 32m
Distance of Load 36 t (54m – 22 m) = 32 m
Distance of Load 48 t (60m – 54 m) = 6m
1 2 3 4 5
Weight (t) Weight (t) Dist. From Head Stern
LCF (m) moments moments
(t-m) (t-m)
120 (L) 120 44 m 5,280
68 (L) 68 32 m 2,176
- 36 (D) 36 32 m 1,152
- 48 (D) 48 6m 288
104 Load 3,328 5,568
2,240
b. Calculate bodily sinkage:
w
Bodily Sinkage=
TPC
104 t
Bodily Sinkage=
14.2t /cm
Bodily Sinkage= 7. 3 cm (convert to meters)

Bodily Sinkage= 0.073 m

@ All Rights Reserved


Issue No. 1 Page 159 of 289
ST. THERESE- MTC COLLEGES SEAMANSHIP 6
Iloilo, Philippines (Advance Trim, Stability and Stress)
STUDENT LEARNING MODULE

E
Revision No. 2 Effectivity date: Reviewed by: Approved by:
GEMMA E. PAGSUGIRON, MBA MA. TERESA S. SARABIA, Ph.D.
01 September 2021 QMR President
c. Calculate Change of Trim:
Trimming moments
Change of trim (CT) =
MTC 1C
2240 t
Change of trim (CT) =
166 t−m/cm
Change of trim (CT) = 19.31 cm by the STERN

d. Calculate Change of drafts aft and forward:


l
Change of draft aft = x Change of trim (CT)
L
where: l = 54 m (from aft perpendicular to LCF )
54 m
Change of draft aft = x 19.31 cm
120 m
1,042.7 cm
Change of draft aft =
120
Change of draft aft = 8.68 cm (convert to meters)

Change of draft aft =0.087 m

Change of draft forward =Change of trim (CT) – Change of draft aft


Change of draft forward = 0.193 m – 0.087 m
Change of draft forward = 0.106 m

e. Calculate the Final draughts:


Aft Forward
Initial draughts 6.360 m 6.240 m
Bodily sinkage + 0.073 m + 0.073 m
6.433 m 6.313 m
Change due trim + 0.087m - 0.106 m
New Draughts 6.520 m 6.207 m

Example 2

A ship 220 m in length (LBP) with a TPC of 58 t and MCT1C of 650 t-m while the center of
flotation is 3.8 m forward of amidships. The drafts are 10.57 m forward and 11.00 m aft.
The following cargoes is then worked:
Load 450 t 85 m forward of amidships
Load 1,555 t 43 m abaft of F (Center of flotation)

@ All Rights Reserved


Issue No. 1 Page 160 of 289
ST. THERESE- MTC COLLEGES SEAMANSHIP 6
Iloilo, Philippines (Advance Trim, Stability and Stress)
STUDENT LEARNING MODULE

E
Revision No. 2 Effectivity date: Reviewed by: Approved by:
GEMMA E. PAGSUGIRON, MBA MA. TERESA S. SARABIA, Ph.D.
01 September 2021 QMR President
Discharge 824 t 28 m forward of amidships
Discharge 669 t 35 m abaft of F (Center of flotation)
Illustrate and calculate the final draughts.

220m

669 d=35m
28m
824
.F d=24.2
3.8m
d=43m d=81.2
1555 85m 450

L/2= 110m FP
AP l = 110 + 3.8= 113.8m
Figure 7.20
Solution:
a. Calculate the distances each cargo from F:
Dist. of Loaded weight 450 t (85 m – 3.8 m) = 81.2 m
Dist. of Loaded weight 1,555 t = 43 m
Dist. of Discharged weight 824 t (28 m – 3.8 m) = 24.2 m
Dist. of Discharged weight 669 t = 35 m

1 2 3 4 5
Weight (t) Weight (t) Dist. From Head Stern
LCF (m) moments moments
(t-m) (t-m)
450 (L) 450 81.2 m 36,540
1,555 (L) 1,555 43 m 66,865
- 824 (D) 824 24.2 m 19,940.8
- 669 (D) 669 35 m 23,415
512 Loaded 59,955 86,805.8
26,850.8
b. Calculate bodily sinkage:
w
Bodily Sinkage=
TPC
512t
Bodily Sinkage=
58t / cm
Bodily Sinkage= 8.83 cm (convert to meters)

@ All Rights Reserved


Issue No. 1 Page 161 of 289
ST. THERESE- MTC COLLEGES SEAMANSHIP 6
Iloilo, Philippines (Advance Trim, Stability and Stress)
STUDENT LEARNING MODULE

E
Revision No. 2 Effectivity date: Reviewed by: Approved by:
GEMMA E. PAGSUGIRON, MBA MA. TERESA S. SARABIA, Ph.D.
01 September 2021 QMR President
Bodily Sinkage = 0.09 m

c. Calculate Change of Trim:


Trimming moments
Change of trim (CT) =
MTC 1C
26,850.8 t
Change of trim (CT) =
650 t−m/cm
Change of trim (CT) = 41.31 cm (convert to meters)

Change of trim (CT) = 0.41 m by the STERN

d. Calculate Change of drafts aft and forward:


l
Change of draft aft = x Change of trim (CT)
L
where: l = 54 m (from aft perpendicular to LCF )
113.8m
Change of draft aft = x 0.41 m
220 m

46.658 m
Change of draft aft =
220
Change of draft aft = 0.21 m

Change of draft forward =Change of trim (CT) – Change of draft aft


Change of draft forward = 0.41 m –0.21 m
Change of draft forward = 0.20 m

e. Calculate the Final draughts:


Aft Forward
Initial draughts 11.10 m 10.57 m
Bodily sinkage + 0.09 m + 0.09 m
11.19 m 10.66 m
Change due trim + 0.21m - 0.20 m
New Draughts 11.40 m 10.46 m

Example 3

A ship 6500 tonnes displacement has a drafts 7.00 m forward and 8.00 m aft. MCT1C is 100
tonnes-m, TPC is 20 tonnes, center of flotation is amidships, 500 tonnes of cargo are then
discharged from each of the following holds (see Figure 7.21).
No. 1 hold, center of gravity 40 m forward of amidships

@ All Rights Reserved


Issue No. 1 Page 162 of 289
ST. THERESE- MTC COLLEGES SEAMANSHIP 6
Iloilo, Philippines (Advance Trim, Stability and Stress)
STUDENT LEARNING MODULE

E
Revision No. 2 Effectivity date: Reviewed by: Approved by:
GEMMA E. PAGSUGIRON, MBA MA. TERESA S. SARABIA, Ph.D.
01 September 2021 QMR President
No. 2 hold, center of gravity 25 m forward of amidships
No. 3 hold, center of gravity 20 m aft of amidships
No. 4 hold, center of gravity 50 m aft of amidships
The following bunkers are also loaded:
140 tonnes at 15 m forward of
60 tonnes at 12 m aft of
Calculate new draughts forward and aft.

d=50m
d=20m .F d=25m
d=40m
500t 500t 500t
500t
60t d=12m 140t d=15m
AP FP
Solution: Figure 7.21
a. Calculate the Net Weight discharged/loaded:
Total cargo discharged = 4 holds x 500 t = 2,000 tonnes
Total bunkers loaded = 140 t + 60 t = - 200 tonnes
Net weight =1,800 tonnes –discharged

b. Calculate bodily sinkage:


w
Bodily Rise =
TPC
1,800t
Bodily Rise= = 90cm = 0.90 meters
20 t /cm
1 2 3 4 5
Weight (t) Weight (t) Dist. From Head Stern
LCF (m) moments moments
(t-m) (t-m)
-500 (D) 500 40 20,000
-500 (D) 500 25 12,500
-500 (D) 500 20 10,000
-500 (D) 500 50 25,000
140 (L) 140 15 2,100
60 (L) 60 12 720

@ All Rights Reserved


Issue No. 1 Page 163 of 289
ST. THERESE- MTC COLLEGES SEAMANSHIP 6
Iloilo, Philippines (Advance Trim, Stability and Stress)
STUDENT LEARNING MODULE

E
Revision No. 2 Effectivity date: Reviewed by: Approved by:
GEMMA E. PAGSUGIRON, MBA MA. TERESA S. SARABIA, Ph.D.
01 September 2021 QMR President
1,800 Discharged 37,100 33,220
3,880

c. Calculate Change of Trim:


Trimming moments
Change of trim (COT) =
MTC 1C
3,880 t
Change of trim (COT) =
100t−m/cm
Change of trim (COT) = 38.8 cm by the HEAD

d. Calculate Change of drafts aft and forward:


Note: Since the center of flotation is amidships,

Change of draft aft = Change of draft forward


1
=change of trim
2
38.8 cm
Change of draft aft = = 19.4 cm = 0.19 m
2

e. Calculate the Final draughts:


Aft Forward
Initial draughts 8.00 m 7.00 m
Bodily rise - 0.90 m - 0.90 m
7.10 m 6.10 m
Change due trim - 0.19 m + 0.19 m
New Draughts 6.91 m 6.29 m

Do This

Activity 1 – Trim Calculation Multiple Weights


A ship if floating at drafts of 7.10 meters forward and 7.70 meters. The following cargo is
loaded in the flowing holds:

No. 1 hold, 20 tonnes, center of gravity 30 m forward of amidships


No. 2 hold, 45 tonnes, center of gravity 25 m forward of amidships
No. 3 hold, 60 tonnes, center of gravity 15 m aft of amidships

@ All Rights Reserved


Issue No. 1 Page 164 of 289
ST. THERESE- MTC COLLEGES SEAMANSHIP 6
Iloilo, Philippines (Advance Trim, Stability and Stress)
STUDENT LEARNING MODULE

E
Revision No. 2 Effectivity date: Reviewed by: Approved by:
GEMMA E. PAGSUGIRON, MBA MA. TERESA S. SARABIA, Ph.D.
01 September 2021 QMR President
No. 4 hold, 30 tonnes, center of gravity 3 m aft of amidships
The center of floatation is amidships, MCT1C 200 t-m and TPC 35 tonnes. Calculate the new
drafts forward and aft.

Do This

Activity 2 – Trim Calculation Multiple Weights

A ship 100 m long has a center of flotation 3.00 meters aft of amidships and is floating at
drafts 3.20 m forward and 4.40 meters aft. A TPC 10 tonnes, MCT1C 150 t-m. A 30 tonnes
of cargo is discharged from 20 m forward of amidships and 40 tonnes is discharged from 12
m aft of amidships. Calculate the new final drafts.

Do This

Activity 3 – Trim Calculation – Removing Weights

A vessel floats at drafts of 5.12 m forward and 4.69 m aft. Her T.P.C. is 14.4, MCT1C IS 102
and F is amidships. What will be the new drafts after 98 tonnes of water has been pumped
out of the fore peak tank, 52 meters forward of amidships?

Assessment 1 – Trim Calculation Multiple Weights

A ship arrives Iloilo International Port with drafts 6.80 m forward and 7.20 m aft. 500 tonnes
of cargo is then discharged from each of 4 holds.

The center of gravity of No. 1 hold is 40 m forward of amidships


The center of gravity of No. 2 hold is 25 m forward of amidships
The center of gravity of No. 3 hold is 20 m aft of amidships
The center of gravity of No. 4 hold is 50 m aft of amidships

@ All Rights Reserved


Issue No. 1 Page 165 of 289
ST. THERESE- MTC COLLEGES SEAMANSHIP 6
Iloilo, Philippines (Advance Trim, Stability and Stress)
STUDENT LEARNING MODULE

E
Revision No. 2 Effectivity date: Reviewed by: Approved by:
GEMMA E. PAGSUGIRON, MBA MA. TERESA S. SARABIA, Ph.D.
01 September 2021 QMR President
Also, 50 tonnes of cargo is loaded in a position whose center of gravity is 15.00 m aft of
amidships and 135 tonnes of cargo center of gravity 40 m forward of amidships. TPC 15
tonnes, MCT1C is 400 t-m. The center of flotation is amidships. Calculate the final drafts.

Assessment 2 – Trim Calculation Multiple Weights

MV St Therese is 140 meters long, has a TPC 0f 20 and an MCT1C of 120, while the center
of flotation is 3.00 meters abaft amidships. The draft is 7.10 meters forward and 7.25
meters aft. The following cargo is then worked:

Loaded 120 tonnes, 50 meters forward of F.


Loaded 70 tonnes, 20 meters abaft F.
Discharged 90 tonnes , 30 meters forward of F.

Calculate the new displacement and new drafts.

@ All Rights Reserved


Issue No. 1 Page 166 of 289
ST. THERESE- MTC COLLEGES SEAMANSHIP 6
Iloilo, Philippines (Advance Trim, Stability and Stress)
STUDENT LEARNING MODULE

E
Revision No. 2 Effectivity date: Reviewed by: Approved by:
GEMMA E. PAGSUGIRON, MBA MA. TERESA S. SARABIA, Ph.D.
01 September 2021 QMR President

Lesson 8: Curve of Statical Stability

Learning Module 8.1: Curve of Statical Stability (Week 9, 6 hours)

Competence, Course Outcomes and Learning Outcomes

Competence:

Control trim, stability and stress

Course Outcome:

1. Explain the importance of maintaining stability during loading, unloading


and in-transit in various conditions.
2. Calculate the effect on trim and stability of a ship in the event of damage
to and consequent flooding of a compartment and countermeasures to be
taken.

Learning Outcomes:

At the end of the learning module, the student can:

1. Construct a curve of statical stability and


2. Calculate the area under the curve in meter-radians up to a stated angle using
Simpson’s Rules

Introduction

The curve of statical stability, or GZ curve as it is most commonly referred to, is a graphical
representation of the ship’s transverse statical stability.

Transverse statical stability is the term used to describe the ability of a ship to return to the
upright, when it has been forcibly heeled by an external force and is and is momentarily at
rest when floating in still water.

Righting Moment (t-m) = GZ (M) X Displacement

At any angle of heel, it is the horizontal disposition of G and B that determines the GZ value.

@ All Rights Reserved


Issue No. 1 Page 167 of 289
ST. THERESE- MTC COLLEGES SEAMANSHIP 6
Iloilo, Philippines (Advance Trim, Stability and Stress)
STUDENT LEARNING MODULE

E
Revision No. 2 Effectivity date: Reviewed by: Approved by:
GEMMA E. PAGSUGIRON, MBA MA. TERESA S. SARABIA, Ph.D.
01 September 2021 QMR President
As the ship progressively heels over the righting lever, GZ, increases to some maximum
value and then decreases until at some angle of heel it becomes negative i.e. it becomes a
capsizing lever.

Calculating the value of GZ at specified angles of heel for a ship’s particular condition of
loading, will allow a curve of statical stability, or GZ curve, to be produced.

The greater the value of GZ, the greater will be the area under the curve. Minimum
standards with respect to the area under the curve (and other criteria) are specified in the
“Code on Intact Stability (IMO)” and these are incorporated in the government legislation of
most countries that adopt the IMO conventions.

It is aim of this module to review the method of actually producing a curve of statical
stability and be able to extract basic information from it.

8.1 Curves of Statical Stability (GZ Curves)

In order to illustrate the length of the righting arms at each angle, these levers are
displayed as a curve, the stability curve. The curve shows the stability of the ship at heeling
angles from zero upwards (usually up to 60°).

The curve of the righting arms applies to a specific draft and weight distribution. Any other
condition gives another curve.

The righting arm must be sufficiently large at each angle for the ship to right itself during
various conditions such as bad weather.

The curve must be derived and evaluated:

 During loading or discharging and before sailing


 For changes of weight distribution during the voyage due to consumption of fuel
and/or drinking water. This has to be considered before leaving port.

The calculation and appraisal of the curve can be produced by computer.

With each new entry the ship’s loading computer immediately recalculates the righting arms
and thus, the curve.

The responsible officers must be thoroughly aware of the basis of this calculation. They
must know which rules to apply if the ships stability decreases to such an extent that action
is required.

@ All Rights Reserved


Issue No. 1 Page 168 of 289
ST. THERESE- MTC COLLEGES SEAMANSHIP 6
Iloilo, Philippines (Advance Trim, Stability and Stress)
STUDENT LEARNING MODULE

E
Revision No. 2 Effectivity date: Reviewed by: Approved by:
GEMMA E. PAGSUGIRON, MBA MA. TERESA S. SARABIA, Ph.D.
01 September 2021 QMR President
Remarks about stability curve:

– As long as the weight of the ship does not change, the shape of the curve is
determined by the ships form and the water-plane area. With a small freeboard or
small draft the water-plane area can change considerably under heel.
– The curve is only applicable if the ship lies in calm water. If the ship sails in waves
the water-plane area changes and thus, a continuously changing curve results.
– The curve shows:
 the righting moment or the righting arm at each angle of heel;
 the energy produced by the righting moment to resist a heel from 0° to any
chosen angle
– The magnitude of the levers and thus the area under the curve, will decrease if:
 weight (cargo, ballast, etc.) is placed above G;
 weight is discharged under G.
– Changes in the direction of the GZ curve are caused when:
 The deck edge is submerged
 The coaming is submerged
 The bilge rises above water

These are all due to the waterline breadth changing positively or negatively.
– The horizontal scale is the distance calculated from 0° to the point where the
righting arm is negative (more than 83°). That is, the point at which the ship will
dynamically overturn.
– The ship will overturn at the top of the curve with a static load.

0.6
0.5

0.4 Deck Immersion

0.3

0.2

0.1

0.0 10.0 20.0 30.0 40.0 50.0 60.0 70.0 80.0 90.0
Heeling angle (Degrees)
Figure 8.1
= point of inflection

@ All Rights Reserved


Issue No. 1 Page 169 of 289
ST. THERESE- MTC COLLEGES SEAMANSHIP 6
Iloilo, Philippines (Advance Trim, Stability and Stress)
STUDENT LEARNING MODULE

E
Revision No. 2 Effectivity date: Reviewed by: Approved by:
GEMMA E. PAGSUGIRON, MBA MA. TERESA S. SARABIA, Ph.D.
01 September 2021 QMR President

Figure 8.2

8.2 Procedures for Calculating GZ Values


Example 1

MV Almar completes loading with a displacement of 29,000 t and a KG corrected for free
surfaces of 8.92 m. Calculate the GZ values and GM if the KM for the loaded displacement is
9.46 m. (Use extract tabulated KN Values for MV Almar)

Solution:

Using the formula: GZ = KN – (KG Sin θ )

a. Calculate the GZ values for the loaded condition.

To calculate the KG Sin θ :


8.92 m x sin 10° = 1.55 meters
8.92 m x sin 20° = 3.05 meters
8.92 m x sin 30° = 4.46 meters
8.92 m x sin 40° = 5.73 meters
8.92 m x sin 60° = 7.72 meters
8.92 m x sin 80° = 8.78 meters

Heel 10° 20° 30° 40° 60° 80°


KN 1.65 3.31 4.93 6.30 7.86 8.07
KG* Sin Heel - 1.55 - 3.05 - 4.46 - 5.73 - 7.72 - 8.78

GZ (arm) 0.10 0.26 0.47 0.57 0.14 - 0.71

Table 1: KN Values Table

@ All Rights Reserved


Issue No. 1 Page 170 of 289
ST. THERESE- MTC COLLEGES SEAMANSHIP 6
Iloilo, Philippines (Advance Trim, Stability and Stress)
STUDENT LEARNING MODULE

E
Revision No. 2 Effectivity date: Reviewed by: Approved by:
GEMMA E. PAGSUGIRON, MBA MA. TERESA S. SARABIA, Ph.D.
01 September 2021 QMR President

b. Calculate the GM: KM 9.460 m


GM = KM - KG
KG ~ 8.920 m
GM 0.54 m

8.3 Procedures for Constructing the Curve of Statical Stability

The following steps should always be undertaken when producing a curve of statical
stability.

1. Determine the ship’s displacement and effective KG for the condition being
considered (effective KG being that taking into account free liquid surfaces in tanks).
2. From the hydrostatic data find the value of KM for the ship’s displacement.
3. Find the GMFLUID using:
GM = KM – KGFLUID

@ All Rights Reserved


Issue No. 1 Page 171 of 289
ST. THERESE- MTC COLLEGES SEAMANSHIP 6
Iloilo, Philippines (Advance Trim, Stability and Stress)
STUDENT LEARNING MODULE

E
Revision No. 2 Effectivity date: Reviewed by: Approved by:
GEMMA E. PAGSUGIRON, MBA MA. TERESA S. SARABIA, Ph.D.
01 September 2021 QMR President
4. Enter KN tables (or curves) and obtain KN value in meters for each angle of heel
given.
5. Using: GZ = KN – (KG Sin θ ) determine the GZ values for the angles of heel
given.
6. Plot the GZ values
7. Before joining all points on the curve construct a vertical at 5.73°and from the base
upwards mark off the value of the effective GM (using the GZ scale). From this point
draw a straight line to the origin of the curve to be drawn. This will indicate the
initial trend of the curve at small angles of heel and will assist in sketching the actual
curve between the origin and the first plotted GZ value.

(GZ and GM are closely related at small angles of heel)

The curve for the GZ values calculated in the example is shown. Note the construction using
the initial GM value of 0.54m.

Heel 10 20 30 40 60 80
KN 1.65 3.31 4.93 6.30 7.86 8.07
KG* Sin Heel 1.55 3.05 4.46 5.73 7.72 8.78
GZ (arm) 0.10 0.26 0.47 0.57 0.14 - 0.71

0.54

57.3

Heel (deg.)

Figure 8.3 Curve of static stability

@ All Rights Reserved


Issue No. 1 Page 172 of 289
ST. THERESE- MTC COLLEGES SEAMANSHIP 6
Iloilo, Philippines (Advance Trim, Stability and Stress)
STUDENT LEARNING MODULE

E
Revision No. 2 Effectivity date: Reviewed by: Approved by:
GEMMA E. PAGSUGIRON, MBA MA. TERESA S. SARABIA, Ph.D.
01 September 2021 QMR President
Example 2

MV St. Therese completes loading with a displacement of 3,500 t and a KG of 4.9 m.


Calculate the GZ values and draw the stability curve for this loading condition.

ANGLE OF HEEL - DEGREES


DISPLACEMENT
10° 20° 30° 40° 50° 60°
(t)
4500 0.894 1.782 2.611 3.453 4.103 4.552

4250 0.891 1.791 2.630 3.489 4.150 4.552

4000 0.890 1.793 2.658 3.521 4.192 4.584

3750 0.892 1.799 2.699 3.551 4.228 4.614


3500 0.919 1.862 2.853 3.672 4.257 4.684
3250 0.905 1.831 2.804 3.626 4.277 4.666

Figure 8.4: Cross Curves of Stability (KN Values)

Heeling angle 10° 20° 30° 40° 50° 60°


KN 0.919 1.862 2.853 3.672 4.257 4.684

KG* Sin Heel 0.851 1.676 2.450 3.150 3.754 4.244


GZ (arm) 0.068 0.186 0.403 0.522 0.555 0.440

Deck Immersion

GMO

Heeling angle (Degrees)


Figure 8.5: Curves of Static Stability

@ All Rights Reserved


Issue No. 1 Page 173 of 289
ST. THERESE- MTC COLLEGES SEAMANSHIP 6
Iloilo, Philippines (Advance Trim, Stability and Stress)
STUDENT LEARNING MODULE

E
Revision No. 2 Effectivity date: Reviewed by: Approved by:
GEMMA E. PAGSUGIRON, MBA MA. TERESA S. SARABIA, Ph.D.
01 September 2021 QMR President
8.4 Information Available from the Curve of Statical Stability
Based on the curve (Figure 8.3) the following information can be obtained from it.

1. The GZ value for any angle of heel;


This can be used to calculate the moment of statical stability for the ship at that
particular angle of heel if the formula”
Righting moment (t-m) = GZ (m) x Displacement (t) is applied.
2. The maximum GZ and the angle of heel at which it occurs.
3. The range of positive stability and the angle of vanishing stability (AVS);
4. The approximate angle of deck edge immersion (θ DEI).

From the curve:

The maximum GZ value is 0.57 m and occurs at an approximate angle of heel of 39°.

θ DEI

Max. GZ

AVS

Range of positive stability

Heel (deg.)

Figure 8.6: Curves of Static Stability

8.5 Determining the area under the curve

The area under the curve represents the energy involved in the dynamic stability. The
problem with calculating the area under the curve is that the levers are given in cm or
meters on the vertical axis and the angles are given in degrees on the horizontal axis.

To overcome this, the heeling angles have to be expressed in terms of distance. This is done
by converting the heeling angles to radians, by dividing the number of heeling degrees by
57.3°. When the area below the curve is so calculated, centimeter radians or meter radians
are obtained.

@ All Rights Reserved


Issue No. 1 Page 174 of 289
ST. THERESE- MTC COLLEGES SEAMANSHIP 6
Iloilo, Philippines (Advance Trim, Stability and Stress)
STUDENT LEARNING MODULE

E
Revision No. 2 Effectivity date: Reviewed by: Approved by:
GEMMA E. PAGSUGIRON, MBA MA. TERESA S. SARABIA, Ph.D.
01 September 2021 QMR President
The area under the curve can now be calculated in the following ways:

– Make a triangle with the same area as the area under the curve. This is less
accurate, but still a good way to obtain a quick impression of the area (surface
area= ½ x base x height). This can be applied to regulation a1, a2 and a3.
– Make a rectangle of the area. (surface area = base x height).
This can be applied to regulations b1 and b2.
– Using the Simpson’s Rules

The area under the stability curve, representing the energy a ship can exert against a
moment caused by a list, can be represented as ΣGZφ , whereby φ is reported in radians.

As long as the curve runs evenly, the calculation of the area can be estimated by regarding
the area 0 – 30° as a triangle and the area 30 – 40°, as trapezium.

Area 0 – 30° is then: 0.5 x (30°/57.3°) x GZ 30° mrad = 0.262 x GZ 30° mrad

Area 0 – 40° is then: 0.5 x (30°/57.3°) x (GZ 30° + GZ 40°) mrad = 0.873 x GZ 30°+GZ
40 °) mrad
For more accurate calculation of the area, especially if the curve is less regular, such as
ships with low freeboard and a high coaming, Simpson’s Rules can be used assuming that
the curve is of the second degree.

The substantiation of this will not be addressed here. The resulting area is a reasonably
accurate estimate.

The practical application is (see figure 1 and 2):

The curve is divided into a number of equal parts (h) on the X axis, in this case four,
resulting in an even number of vertical ordinates (Y 0-Y4).

According to Simpson’s 1st rule, the area is now:


1
x h x (Y0 + 4 x Y1 + 2 + Y2 + 4 x Y3 + Y4).
3

Y Y1 Y3 Y4
Y0 Y2
a-
s
h
Figure 8.7 X-as

GZ 10° 20° 30° 40° 50°


@ 60°
All Rights Reserved
Issue No. 1 Page 175 of 289
ST. THERESE- MTC COLLEGES SEAMANSHIP 6
Iloilo, Philippines (Advance Trim, Stability and Stress)
STUDENT LEARNING MODULE

E
Revision No. 2 Effectivity date: Reviewed by: Approved by:
GEMMA E. PAGSUGIRON, MBA MA. TERESA S. SARABIA, Ph.D.
01 September 2021 QMR President

GZ10 GZ20 GZ30 GZ40

10° =
0.175 rad Figure 8.8

The calculation of the area under the curve using the 1 st rule of Simpson works as follows:

– The area from 0° - 30°, 0° - 40° and the increase from 30° - 40° must be calculated.
– There is one problem in calculating the area 0 - 30°.
– Because this part of the curve cannot be divided in an equal number of parts on the
X axis (from which the GZ value is determined), an intermediate step is used.

First, the area 0° - 10° has to be calculated and this part of the curve regarded as a triangle
(area shaded in blue).

Next, the area from 10° - 30° (in red) has to be calculated; together, they indicate the area
0° - 30°.

The area from 0° - 40° can be directly figured using Simpson’s 1 st Rule.

To reduce the area from 0° - 30°, an increase from 30° - 40° is obtained (in green)10° is
equal to 10°/57.3° = 0.175 radians.

The calculation now includes the following steps:

Area 0° - 10° = 0.5 x 0.175 x GZ1 = A

1
Area 10°-30° = x 0.175 x (GZ 10° + 4 x GZ 20° + GZ 30°) = B
3
Area 0° - 30° = A + B

@ All Rights Reserved


Issue No. 1 Page 176 of 289
ST. THERESE- MTC COLLEGES SEAMANSHIP 6
Iloilo, Philippines (Advance Trim, Stability and Stress)
STUDENT LEARNING MODULE

E
Revision No. 2 Effectivity date: Reviewed by: Approved by:
GEMMA E. PAGSUGIRON, MBA MA. TERESA S. SARABIA, Ph.D.
01 September 2021 QMR President
1
Area 0° - 40° = x 0.175 x (GZ 0° + 4 x GZ 10° + 2 x GZ 20° + 4 x GZ 30° + GZ 40°)=C
3
Increase 30° - 40° = C – (A + B).

We need a method for determining the areas under parts of the GZ curve, and hence the
dynamical stability between two heel angles. A fairly accurate method of doing this is by
using “Simpson’s Rules.

An “ordinate” is the distance from a fixed reference line. In this case the reference line is
the horizontal heel axis. The GZ curve is then defined using ordinates at regular intervals. In
the above diagram the interval (h) is 10°. However, degrees of arc are of little use when
calculating an area, so it is necessary to convert degrees into radians (a linear measure of
arc as well as another method of measuring an angles). One radian is the equivalent of 57.3
degrees of arc, therefore to convert degrees into radians we must divide the angle in
degrees by 57.3.

10
For example 10° = radians = 0.17452 radians
57.3

Example 1:

A ship’s Curve of Statical Stability has the following ordinates:


0°=0 10°=0.20m 20°=0.52m 30°=0.95m 40°=1.22m 50°=1.16m 60°=0.75m

Construct curve of statical stability and calculate the area under the curve using the
Simpson’s Rule.

1. Plot the GZ values

Figure 8.9
2. Join all points on the curve

@ All Rights Reserved


Issue No. 1 Page 177 of 289
ST. THERESE- MTC COLLEGES SEAMANSHIP 6
Iloilo, Philippines (Advance Trim, Stability and Stress)
STUDENT LEARNING MODULE

E
Revision No. 2 Effectivity date: Reviewed by: Approved by:
GEMMA E. PAGSUGIRON, MBA MA. TERESA S. SARABIA, Ph.D.
01 September 2021 QMR President

h = 10°

Figure 8.10
There are 6 common interval (h).

AREA UNDER
THE CURVE

Figure 8.11

Solution: Use Simpson’s First Rule.

@ All Rights Reserved


Issue No. 1 Page 178 of 289
ST. THERESE- MTC COLLEGES SEAMANSHIP 6
Iloilo, Philippines (Advance Trim, Stability and Stress)
STUDENT LEARNING MODULE

E
Revision No. 2 Effectivity date: Reviewed by: Approved by:
GEMMA E. PAGSUGIRON, MBA MA. TERESA S. SARABIA, Ph.D.
01 September 2021 QMR President
10
Common Interval (h) = 10° = radians
57.3

1
Formula: Area = x h x (1a + 4b + 2c + 4d + 2e + 4f + 1g)
3
1
or Area = h (Sum of Products for Area)
3

Heel (°) Ordinate (m) Simpson’s Products for Area


Multiplier
0° 0 1 0
10° 0.20 4 0.80
20° 0.52 2 1.04
30° 0.95 4 3.80
40° 1.22 2 2.44
50° 1.16 4 4.64
60° 0.76 1 0.76
Sum of product for 13.48
area (Σ1)

1
Area = h (Sum of Products for Area)
3

1 10
Area = x x 13.48
3 57.3

10 x 13.48 134.8
Area = = = 0.7842 meter-radians
3 x 57.3 171.9

Do This
@ All Rights Reserved
Issue No. 1 Page 179 of 289
ST. THERESE- MTC COLLEGES SEAMANSHIP 6
Iloilo, Philippines (Advance Trim, Stability and Stress)
STUDENT LEARNING MODULE

E
Revision No. 2 Effectivity date: Reviewed by: Approved by:
GEMMA E. PAGSUGIRON, MBA MA. TERESA S. SARABIA, Ph.D.
01 September 2021 QMR President

Activity 1 – Curve of Statical Stability

MV St Prestige completes loading with a displacement of 19,000 t and a KG corrected for


free surfaces of 8.85 m. KM for the loaded displacement is 9.48 m. (Use the extract
tabulated KN Values)

1. Calculate the GZ values.


2. Calculate the area under the curve from 0º to 40º using the Simpson’s Rule.
3. Construct curve of statical stability using a graphing paper.

Assessment – Curve of Statical Stability

@ All Rights Reserved


Issue No. 1 Page 180 of 289
ST. THERESE- MTC COLLEGES SEAMANSHIP 6
Iloilo, Philippines (Advance Trim, Stability and Stress)
STUDENT LEARNING MODULE

E
Revision No. 2 Effectivity date: Reviewed by: Approved by:
GEMMA E. PAGSUGIRON, MBA MA. TERESA S. SARABIA, Ph.D.
01 September 2021 QMR President
MV St Therese completed loading with a displacement of 31,000 t and a KG corrected for
free surfaces of 7.54 m. KM for the loaded displacement is 8.10 m. (Use your extract
tabulated KN Values).

1. Calculate the GZ values.


2. Calculate the area under the curve using the Simpson’s Rule.
3. Construct curve of statical stability using a graphing paper.

Lesson 9: Dynamical Stability

Learning Module 9.1: Dynamical Stability (Week 10, 6 hours)

@ All Rights Reserved


Issue No. 1 Page 181 of 289
ST. THERESE- MTC COLLEGES SEAMANSHIP 6
Iloilo, Philippines (Advance Trim, Stability and Stress)
STUDENT LEARNING MODULE

E
Revision No. 2 Effectivity date: Reviewed by: Approved by:
GEMMA E. PAGSUGIRON, MBA MA. TERESA S. SARABIA, Ph.D.
01 September 2021 QMR President
Competence, Course Outcomes and Learning Outcomes

Competence:

Control trim, stability and stress

Course Outcome:

1. Explain the importance of maintaining stability during loading, unloading


and in-transit in various conditions.
2. Calculate the effect on trim and stability of a ship in the event of damage
to and consequent flooding of a compartment and countermeasures to be
taken.

Learning Outcomes:

At the end of the lesson, the student should be able to:

1. Explain the dynamical stability at a given angle of heel.


2. Calculate the dynamical stability at a given angle of heel.

Figure 9.1

Discussion

9.1 Definition

@ All Rights Reserved


Issue No. 1 Page 182 of 289
ST. THERESE- MTC COLLEGES SEAMANSHIP 6
Iloilo, Philippines (Advance Trim, Stability and Stress)
STUDENT LEARNING MODULE

E
Revision No. 2 Effectivity date: Reviewed by: Approved by:
GEMMA E. PAGSUGIRON, MBA MA. TERESA S. SARABIA, Ph.D.
01 September 2021 QMR President
The ‘dynamical stability” of a ship at any particular angle of inclination may be defined as
‘the work required by the external forces (wind, waves etc.) to heel the ship to that angle’ of
heel.

9.2 Work

Suppose that we wish to push a weight across the deck of a ship. The weight will resist our
efforts to move it on account of inertia, friction with the deck, etc., and we shall have to
exert force in order to start it moving. If we then stop pushing, the friction between the
deck and the weight will soon cause the latter to stop moving, so we must continue to push
until it is the desired position. The greater the weight, the harder we must push and the
greater the distance, the longer we must push. In other words, we must do work and the
amount of work done depends on the distance we have to move the weight and the amount
of force we have to exert in order to move it. Thus, work done is equal to the force exerted,
multiplied by the distance over which it is exerted.
b

θ
W1 g L1
h1
h G g1
Z

B P R L
B1

W
Figure 9.2
9.3 Dynamical Stability

Consider a ship which is being heeled by some external force. As soon as she heels to a
small angle, her moment of statical stability will try to force her back to the upright. In order
to heel her further, sufficient force must be exerted to overcome this statical stability and
must continue to be exerted foe as long as the ship continues to heel. We can liken this
case so that of the weight mentioned in the last paragraph and say that the work done to
heel the ship to any given angle is equal to all the force exerted, over all the distance
through which the ship has heeled. This is obviously only another way of expressing the
definition of dynamical stability, which is given above.

9.4 Dynamical Stability from a Curve of Statical Stability


Figure 9.3 shows a curve of statical stability, in which the moment of statical stability (W x
GZ) is plotted against the angle of heel. The statical stability at any angle is found by the
perpendicular distance from the base line to the curve at the angle. For instance, the
@ All Rights Reserved
Issue No. 1 Page 183 of 289
ST. THERESE- MTC COLLEGES SEAMANSHIP 6
Iloilo, Philippines (Advance Trim, Stability and Stress)
STUDENT LEARNING MODULE

E
Revision No. 2 Effectivity date: Reviewed by: Approved by:
GEMMA E. PAGSUGIRON, MBA MA. TERESA S. SARABIA, Ph.D.
01 September 2021 QMR President
moment of statical stability at 30° of heel is found by drawing the perpendicular line, AB,
and then the horizontal one, BD. The required moment is then CD – in this case about
13,000 tone-meters.

We have just said that dynamical stability is equal to all the force exerted over all the
distance through which the ship has heeled. This can be taken to mean the sum of all the
moments of statical stability, for every small angle of heel, up to the given angle.

D
B

Figure 9.3

Figure 9.4

Consider Figure 9.4, we shall see that the sum of all the moments of statical stability up to
30° of heel will be equal to the shaded area ABC. Similarly for other angle. This means that
the dynamical stability at any angle is equal to the area under the curve of statical stability
up to that angle. For this purpose, the vertical distances to the curve are always measured
in terms of statical stability and the length along the base line in terms of circular measure
(or “radians”). A radian is equal to 57.3°, so that the length along the base line becomes:

@ All Rights Reserved


Issue No. 1 Page 184 of 289
ST. THERESE- MTC COLLEGES SEAMANSHIP 6
Iloilo, Philippines (Advance Trim, Stability and Stress)
STUDENT LEARNING MODULE

E
Revision No. 2 Effectivity date: Reviewed by: Approved by:
GEMMA E. PAGSUGIRON, MBA MA. TERESA S. SARABIA, Ph.D.
01 September 2021 QMR President
Angle of heel(°)
57.3

Thus, given a scale or curve of righting levers or moments, we can use Simpson’s Rules to
find the Dynamical Stability.

We should always use moments of statical stability (W x GZ) to obtain dynamical stability,
which would be measured in “tonne meter-radians”. For the purpose of comparison,
however, we often use righting levers (GZ), to give a quantity called “meter radians”. The
latter can be converted to actual dynamical stability by multiplying it by the ship’s
displacement (W).

Therefore, the dynamically stability to any angle of heel is found by multiplying the area
under the stability curve to that angle by the displacement.

Formula:
Dynamical stability = W x Area under the stability curve

It should be noted that in finding the area under the stability curve by the use of Simpson’s
Rules, the common interval must be expressed in radians:

57.3° = 1 radian

1
1° = radians
57.3

x
or x° = radians
57.3

Therefore, to convert degrees to radians, simply divide the number of degrees by 57.3.

Process me
Example 1

@ All Rights Reserved


Issue No. 1 Page 185 of 289
ST. THERESE- MTC COLLEGES SEAMANSHIP 6
Iloilo, Philippines (Advance Trim, Stability and Stress)
STUDENT LEARNING MODULE

E
Revision No. 2 Effectivity date: Reviewed by: Approved by:
GEMMA E. PAGSUGIRON, MBA MA. TERESA S. SARABIA, Ph.D.
01 September 2021 QMR President
Calculate the dynamical stability at 40° heel, for a ship which displaces 6,600 tonnes and
which has the following righting levers:

Angle of 10° 20° 30° 40°


Heel

GZ (m) 0.18 0.41 0.67 0.85


Figure 9.4
10
Common interval (h) = 10° = radians
57.3

Inclination (°) GZ (m) / Simpson’s Area Product


Ordinate Multiplier
0 0.00 1 0.00
10 0.18 4 0.72
20 0.41 2 0.82
30 0.67 4 2.68
40 0.85 1 0.85
Sum of product for
area (Σ1) 5.07

1
Area = h (Sum of Products for Area)
3

1 10
Area = x x 5.07
3 57.3

10 x 5.07 50.7
Area = = = 0.2949 meter-radians
3 x 57.3 171.9

Dynamical stability = W x Area under the stability curve

= 6,600 t x 0.2949 meter-radians

Dynamical stability = 1,946.34 tonne meter-radians

Example 2

Illustrate and calculate the dynamical stability at 40° heel, for a ship which displaces 5,200
tonnes and which has the following righting levers:

@ All Rights Reserved


Issue No. 1 Page 186 of 289
ST. THERESE- MTC COLLEGES SEAMANSHIP 6
Iloilo, Philippines (Advance Trim, Stability and Stress)
STUDENT LEARNING MODULE

E
Revision No. 2 Effectivity date: Reviewed by: Approved by:
GEMMA E. PAGSUGIRON, MBA MA. TERESA S. SARABIA, Ph.D.
01 September 2021 QMR President
Heel 0° 10° 20° 30° 40°

GZ (m) 0 0.21 0.35 0.45 0.48

Inclination (°) GZ (m) / Simpson’s Area Product


Ordinate Multiplier
0 0.00 1 0.00
10 0.21 4 0.84
20 0.35 2 0.70
30 0.45 4 1.80
40 0.48 1 0.48
Sum of product for
area (Σ1) 3.82
Solution:
1
Area under the stability curve = h (Sum of Products for Area)
3
1 10
Area = x x 3.82
3 57.3

10 x 3.82 38.2
Area = = = 0.2222 meter-radians
3 x 57.3 171.9

Dynamical stability = W x Area under the stability curve

= 5,200 t x 0.2222 meter-radians

Dynamical stability = 1,155.44 tonne meter-radians

Example 3

A box-shaped vessel 55 m x 15 m x 8 m is floating in salt water at a draft of 5.00 m forward


and aft. The GM is 0.70. Calculate the dynamical stability to 20° heel.
@ All Rights Reserved
Issue No. 1 Page 187 of 289
ST. THERESE- MTC COLLEGES SEAMANSHIP 6
Iloilo, Philippines (Advance Trim, Stability and Stress)
STUDENT LEARNING MODULE

E
Revision No. 2 Effectivity date: Reviewed by: Approved by:
GEMMA E. PAGSUGIRON, MBA MA. TERESA S. SARABIA, Ph.D.
01 September 2021 QMR President
Formula: Displacement = L x B x Draft
B2
BM =
12d
GZ = GM x Sin θ

Solution:

1. Displacement = L x B x Draft
Displacement = 55 m x 15 m x 5.00 m x 1.025 t/m3
Displacement = 4,228 tonnes

B2
2. BM = 12d

15 m x 15 m
BM =
12 x 5 m

225 m2
BM = = 3.75 m
60 m

Note: When calculating the GZs 10° may be considered a small angle of heel, but 20° is a
large angle of heel and therefore the wall sided formula must be used to find the GZ.

3. GZ ordinates are derived below:

At 10° heel: GZ = GM x Sin θ


GZ = 0.7 m x Sin 10°
GZ = 0.7 m x 0.17365 (R.O. to 5 decimal)
GZ = 0.122 meters (R.O. to 3 decimal)
1
At 20° heel: GZ = (GM + BM tan2θ ) Sin θ
2
1
= (0.7 + x 3.75 tan2 20°) Sin 20°
2
1
= (0.7 + x 3.75 x 0.363972) Sin 20°
2
1
= (0.7 + x 3.75 x 0.13247) (0.34202)
2
0.49676
= (0.7 + ) (0.34202)
2
= (0.7 + 0.24838) (0.34202)

@ All Rights Reserved


Issue No. 1 Page 188 of 289
ST. THERESE- MTC COLLEGES SEAMANSHIP 6
Iloilo, Philippines (Advance Trim, Stability and Stress)
STUDENT LEARNING MODULE

E
Revision No. 2 Effectivity date: Reviewed by: Approved by:
GEMMA E. PAGSUGIRON, MBA MA. TERESA S. SARABIA, Ph.D.
01 September 2021 QMR President
= (0.94838 x 0.34202)

GZ= 0.324 m (R.O. to 3 decimal)

Inclination (°) GZ (m) / Simpson’s Area Product


Ordinate Multiplier
0 0.000 1 0.00
10 0.122 4 0.488
20 0.324 1 0.324
Sum of product for
area (Σ1) 0.830
1
Area under the stability curve = h (Σ1)
3

1 10
Area = x x 0.830
3 57.3

10 x 0.830 m 8.30
Area = = = 0.04828 meter-radians
3 x 57.3 171.9

Dynamical stability = W x Area under the stability curve

= 4,228 t x 0.04828 meter-radians

Dynamical stability = 204.13 tonne meter-radians

Dynamical stability is important in ship stability for two reasons. Since it is the measure of
the work that must be done to heel a ship:

1. It is a big factor in deciding how a ship will roll, in this case the waves are doing the
work;

2. It determines the ability of a sailing ship to stand-up under sail; the pressure of the
wind on the sails supplying the work in this case. This is not of much interest to the
average merchant seaman today, but is important to yachtsmen and other who have
to deal with sailing craft.

It must always be remembered that anything which reduces statical stability will also reduce
dynamical stability. This is yet another count against our old enemy the free surface of
liquids.

Do this

@ All Rights Reserved


Issue No. 1 Page 189 of 289
ST. THERESE- MTC COLLEGES SEAMANSHIP 6
Iloilo, Philippines (Advance Trim, Stability and Stress)
STUDENT LEARNING MODULE

E
Revision No. 2 Effectivity date: Reviewed by: Approved by:
GEMMA E. PAGSUGIRON, MBA MA. TERESA S. SARABIA, Ph.D.
01 September 2021 QMR President
Activity 1: Dynamical Stability Calculation

Calculate the dynamical stability at 40° heel of a ship which displaces 6,876 tonnes and has
the following GZs:

Heel 0° 10° 20° 30° 40°


GZ (m) 0.000 0.115 0.228 0.331 0.429

Do this

Activity 2: Dynamical Stability Calculation

Calculate the dynamical stability at 45° heel of a ship which displaces 5,000 tonnes and has
the following GZs:

Heel 15° 30° 45°


GZ (m) 0.205 0.482 0.611

Assessment – Calculation of Dynamic Stability

Calculate the area under the curve and dynamical stability at 50° heel, for a ship which
displaces 10,000 tonnes displacement and which has the following righting levers when
inclined:

Angle of Heel 0º 10° 20° 30° 40° 50º


GZ (m) 0.0 0.02 0.12 0.21 0.30 0.33

Lesson 10: Intact Stability Code

Learning Module 10.1: Intact Stability Code (Week 11, 6 hours)

Competence, Course Outcomes and Learning Outcomes

@ All Rights Reserved


Issue No. 1 Page 190 of 289
ST. THERESE- MTC COLLEGES SEAMANSHIP 6
Iloilo, Philippines (Advance Trim, Stability and Stress)
STUDENT LEARNING MODULE

E
Revision No. 2 Effectivity date: Reviewed by: Approved by:
GEMMA E. PAGSUGIRON, MBA MA. TERESA S. SARABIA, Ph.D.
01 September 2021 QMR President

Competence:

Control trim, stability and stress

Course Outcome:

1. Explain the importance of maintaining stability during loading, unloading


and in-transit in various conditions.
2. Calculate the effect on trim and stability of a ship in the event of damage
to and consequent flooding of a compartment and countermeasures to be
taken.

Learning Outcomes:

At the end of the learning module, the student can:

1. Explain the purpose of International Code on Intact Stability, 2008 (2000 IS Code)
2. Explain the integration of IMO knowledge on Intact Stability Code 2008 concerning
ship stability

Discussion

10.1 Introduction

This Code has been assembled to provide, in a


single document, mandatory requirements
in the introduction and in part A and @ All Rights Reserved
recommended provisions in part B relating to
intact stability based primarily on existing IMO
Issue No. 1 Page 191 of 289
ST. THERESE- MTC COLLEGES SEAMANSHIP 6
Iloilo, Philippines (Advance Trim, Stability and Stress)
STUDENT LEARNING MODULE

E
Revision No. 2 Effectivity date: Reviewed by: Approved by:
GEMMA E. PAGSUGIRON, MBA MA. TERESA S. SARABIA, Ph.D.
01 September 2021 QMR President

10.2 Purpose

The purpose of the Code is to present mandatory


and recommendatory stability criteria and other
measures for ensuring the safe operation of ships,
to minimize the risk to such ships to the personnel
Figure 10.1
on board and to the environment.

This introduction and part A of the Code address the mandatory criteria and part B contains
recommendations and additional guidelines.

This Code contains intact stability criteria for the following types of ships and other marine
vehicles of 24 m in length and above, unless otherwise stated:
1. cargo ships;
2. cargo ships carrying timber deck cargoes;
3. passenger ships;
4. fishing vessels;
5. special purpose ships;
6. offshore supply vessels;
7. mobile offshore drilling units;
8. pontoons; and
9. cargo ships carrying containers on deck and containerships.

10.2.2 Administrations may impose additional requirements regarding the design aspects of
ships of novel design or ships not otherwise covered by the Code.

The IS Code includes fundamental principles such as:

 general precautions against capsizing (criteria regarding metacentric height (GM) and
righting lever (GZ);
 weather criterion (severe wind and rolling criterion);

@ All Rights Reserved


Issue No. 1 Page 192 of 289
ST. THERESE- MTC COLLEGES SEAMANSHIP 6
Iloilo, Philippines (Advance Trim, Stability and Stress)
STUDENT LEARNING MODULE

E
Revision No. 2 Effectivity date: Reviewed by: Approved by:
GEMMA E. PAGSUGIRON, MBA MA. TERESA S. SARABIA, Ph.D.
01 September 2021 QMR President
 effect of free surfaces and icing; and watertight integrity

The IS Code also addresses related operational aspects like information for the master,
including stability and operating booklets and operational procedures in heavy weather.

10.3 General Criteria

10.3.1 Other ships stability criteria

Criteria regarding righting lever curve properties:

1. The area under the righting lever curve (GZ curve) shall not be less
than 0.055 meter-radians up to φ = 30° angle of heel and not less than 0.09 meter-
radians up to φ = 40° or the angle of down-flooding φ if this angle is less than 40°.
Additionally, the area under the righting lever curve (GZ curve) between the angles
of heel of 30° and 40° or between 30° and φ , if this angle is less than 40°, shall not
be less than 0.03 meter-radians.
2. The righting lever GZ shall be at least 0.2 m at an angle of heel equal to or greater
than 30°.
3. The maximum righting lever shall occur at an angle of heel not less than 25°. If this
is not practicable, alternative criteria, based on an equivalent level of safety, may be
applied subject to the approval of the Administration.
4. The initial metacentric height GMo shall not be less than 0.15 m.

10.3.2 Additional Criteria for Passenger Ships

5. In addition, the angle of heel on account of crowding of passengers to one side as


defined below shall not exceed 10°.

A minimum weight of 75 kg shall he assumed for each passenger except that this value may
be increased subject to the approval of the Administration. In addition, the mass and
distribution of the luggage shall be approved by the Administration.

The height of the center of gravity for passengers shall be assumed equal to:
1. 1 m above deck level for passengers standing upright. Account may be taken, if
necessary, of camber and sheer of deck; and;
2. 0.3 m above the seat in respect of seated passengers.

6. Passengers and luggage shall be considered to be in the spaces normally at their


disposal, when assessing compliance with the criteria given in 1 to 4.

In addition, the angle of heel on account of turning shall not exceed 10° when calculated
using the following formula:

@ All Rights Reserved


Issue No. 1 Page 193 of 289
ST. THERESE- MTC COLLEGES SEAMANSHIP 6
Iloilo, Philippines (Advance Trim, Stability and Stress)
STUDENT LEARNING MODULE

E
Revision No. 2 Effectivity date: Reviewed by: Approved by:
GEMMA E. PAGSUGIRON, MBA MA. TERESA S. SARABIA, Ph.D.
01 September 2021 QMR President

where:
MR = heeling moment (kNm)
Vo = service speed (m/s)
LWL = length of ship at waterline (m)
= displacement (t)
d = mean draught (m)
KG = height of center of gravity above baseline (m)

As Per Intact Stability Code Part- A Chapter 3.1.

10.3.3 Stability Criteria for Ships Carrying Timber Deck Cargo

Scope

The provisions given hereunder apply to all ships of 24 m in length and over engaged in the
carriage of timber deck cargoes. Ships that are provided with, and make use of, their timber
load line shall also comply with the requirements of regulations 41 to 45 of the 1966 Load
Lines Convention and the Protocol of 1988 relating thereto.
Alternative stability criteria

For ships loaded with timber deck cargoes and provided that the cargo extends
longitudinally between superstructures (where there is no limiting superstructure at the after
end, the timber deck cargo shall extend at least to the after end of the aftermost hatchway)
transversely for the full beam of ship, after due allowance for a rounded gunwale, not
exceeding 4% of the breadth of the ship and/or securing the supporting uprights and which
remains securely fixed at large angles of heel may be:

1. The area under the righting lever curve (GZ curve) shall not be less than 0.08 meter-
radians up to φ = 40° or the angle of flooding if this angle is less than40°.
2. The maximum value of the righting lever (GZ) shall be at least 0.25 m.
3. At all times during a voyage, the metacentric height GM shall not be less than 0.1 m,
taking into account the absorption of water by the deck cargo and/or ice accretion
on the exposed surfaces.
4. When determining the ability of the ship to withstand the combined effects of beam
wind and rolling according to Heeling Weather Criterion (IS Code Chap.2.3, the 16°
limiting angle of heel under action of steady wind shall be complied with, but the
additional criterion of 80% of the angle of deck edge immersion may be ignored.
5. The cargoes extends longitudinally between superstructures.

10.3.4 Stability Criteria for Grain in Bulk

1. The angle of heel due to the shift of the grain is to be not greater than 12 degrees or
the angle at which the deck cargo is immersed, whichever is the lesser.

@ All Rights Reserved


Issue No. 1 Page 194 of 289
ST. THERESE- MTC COLLEGES SEAMANSHIP 6
Iloilo, Philippines (Advance Trim, Stability and Stress)
STUDENT LEARNING MODULE

E
Revision No. 2 Effectivity date: Reviewed by: Approved by:
GEMMA E. PAGSUGIRON, MBA MA. TERESA S. SARABIA, Ph.D.
01 September 2021 QMR President
2. The net or residual area between the heeling arm curve and the righting arm curve
up to the angle of heel of minimum difference between the ordinates of two curves,
or 40 degrees or the angle of flooding ( θ F), whichever is the least, is in all conditions
of loading to be not less than 0.075 meter radians.
3. The initial metacentric height (GM O), after correction for free surface effects of
liquids in tanks, is to be not less than 0.30 m.
4. After loading, the master shall ensure that the ship is upright before proceeding to
sea.
5. Master must demonstrate the ability of the ship at all stages of any voyage to comply
with the stability criteria required by this section.

10.3.5 Containerships greater than 100 m

Application

These requirements apply to containerships greater than 100 m in length as defined in


section 2(Definitions) of the Introduction. They may also be applied to other cargo ships in
this length range with considerable flare or large water plane areas. The Administration may
apply the following criteria instead of those in part A, 2.2.

Intact stability

1. The area under the righting lever curve (GZ curve) should not be less than 0.009/C
meter-radians up toφ = 30° angle of heel, and not less than 0.016/C meter-radians
up to φ = 40° or the angle of flooding φ f(as defined in part A, 2.2 of the IS Code) if
this angle is less than 40°.
2. Additionally, the area under the righting lever curve (GZ curve) between the angles
of heel of 30° and 40° or between 30° and φ f, if this angle is less than 40°, should
not be less than 0.006/C meter-radians.
3. The righting lever GZ should be at least 0.033/C m at an angle of heel equal or
greater than 30°.
4. The maximum righting lever GZ should be at least 0.042/C m.
5. The total area under the righting lever curve (GZ curve) up to the angle of flooding φ
f should not be less than 0.029/C meter-radians.

In the criteria the form factor should be calculated using the formula and figure 10.2:

where:
d = mean draught (m)
D' = moulded depth of the ship, corrected for defined parts of volumes within the
hatch coamings according to the formula:
D = moulded depth of the ship (m);
BD = moulded breadth of the ship (m);
@ All Rights Reserved
Issue No. 1 Page 195 of 289
ST. THERESE- MTC COLLEGES SEAMANSHIP 6
Iloilo, Philippines (Advance Trim, Stability and Stress)
STUDENT LEARNING MODULE

E
Revision No. 2 Effectivity date: Reviewed by: Approved by:
GEMMA E. PAGSUGIRON, MBA MA. TERESA S. SARABIA, Ph.D.
01 September 2021 QMR President
KG = height of the center of mass above base, corrected for free surface effect, not
be taken as less than d (m);
CB = block coefficient;
CW = water plane coefficient;
lH = length of each hatch coaming within Ll4 forward and aft from amidships (m)
(see figure 10.2);
b = mean width of hatch coamings within Ll4 forward and aft from amidships (m)
(see figure 10.2);
h = mean height of hatch coamings within Ll4 forward and aft from amidships (m)
(see figure 10.1);
L = length of the ship (m);
B = breadth of the ship on the waterline (m);
Bm= breadth of the ship on the waterline at half mean draught (m).

b
IH

B/4
D1
D
d KG
d/2

BM
B
BD

lH lH lH

L/4 L/4

L
Figure 10.2

The shaded areas in figure 10.1represent partial volumes within the hatch coamings
considered contributing to resistance against capsizing at large heeling angles when the ship
is on a wave crest 2,3,2,7. The use of electronic loading and stability instrument is
encouraged in determining the ship's trim and stability during different operational
conditions.

Checkpoint
@ All Rights Reserved
Issue No. 1 Page 196 of 289
ST. THERESE- MTC COLLEGES SEAMANSHIP 6
Iloilo, Philippines (Advance Trim, Stability and Stress)
STUDENT LEARNING MODULE

E
Revision No. 2 Effectivity date: Reviewed by: Approved by:
GEMMA E. PAGSUGIRON, MBA MA. TERESA S. SARABIA, Ph.D.
01 September 2021 QMR President
Activity – Identification. Intact Stability Code 2008 Requirement

Directions: Give the corresponding Intact Stability Criteria requirements for different type of
ships based on Intact Stability Code 2008.

_______1. The metacentric height (GM) intact stability criteria requirement for ships
carrying timber deck cargoes
_______2. The initial metacentric height (GM) intact stability criteria requirement for
container ships greater than 100 meters.
_______3. The initial metacentric height (GM) intact stability criteria requirement for
general cargo ships.
_______4. The righting lever GZ intact stability criteria requirement in meters for general
cargo ships.
_______5. The righting lever GZ intact stability criteria requirement in meters for container
ships greater than 100 meters.
_______6. The limiting angle of heel under action of steady wind requirement for ships
carrying timber deck cargoes.
_______7. The maximum righting lever GZ requirement for ships carrying timber deck
cargoes.
_______8. The maximum righting lever GZ requirement in meters for container ships
greater than 100 meters.
_______9. The maximum righting lever GZ requirement in meters for ships carrying timber
deck cargoes.
_______10. The area under GZ curve requirement for ships carrying timber deck cargoes
in meter radians.

Assessment – Intact Stability Code

Multiple Choice: Select the correct answer from the given choices.

1. When was the International Code on Intact Stability, 2008 adopted?


A. 02 December 2008 C. 04 December 2008
B. 04 December 2008 D. 06 December 2008
2. Under what resolution of the IMO can you find information about intact stability
criteria of your vessel?
A. Resolution MSC 75 (69) C. Resolution MSC 269 (85)

@ All Rights Reserved


Issue No. 1 Page 197 of 289
ST. THERESE- MTC COLLEGES SEAMANSHIP 6
Iloilo, Philippines (Advance Trim, Stability and Stress)
STUDENT LEARNING MODULE

E
Revision No. 2 Effectivity date: Reviewed by: Approved by:
GEMMA E. PAGSUGIRON, MBA MA. TERESA S. SARABIA, Ph.D.
01 September 2021 QMR President
B. Resolution MSC 267 (85) D. Resolution MSC 270 (85)
3. When was the International Code for Intact Stability, 2008 entered into force?
A. 01 May 2010 C. 01 July 2010
B. 01 June 2010 D. 01 August 2010

Intact Stability Criteria for General Cargo, Tankers and Passenger Ships

4. What is the general intact stability criteria requirement on the initial GM or


metacentric height?
A. Not less than 0.10 meters C. Not more than 0.10 meters
B. Not less than 0.15 meters D. Not more than 0.15 meters
5. What is the general intact stability criteria requirement on the righting lever GZ?
A. Shall at least 0.10 meters C. Shall at least 0.20 meters
B. Shall at least 0.15 meters D. Shall at least 0.25 meters
6. What is the general intact stability criteria requirement on the righting lever GZ angle
of heel?
A. ≤ 20 degrees C. ≥ 20 degrees
B. ≤ 30 degrees D. ≥ 30 degrees
7. What is the general intact stability criteria requirement on the maximum righting
lever should occur?
A. At heel > 30 preferably but not less than 25 degrees
B. At heel > 30 preferably but not less than 20 degrees
C. At heel > 25 preferably but not less than 30 degrees
D. At heel > 25 preferably but not less than 25 degrees
8. What is the general intact stability criteria requirement on the Area of the GZ curve?
A. Should be at least 0.055 m radian up to 10 degrees
B. Should be at least 0.055 m radian up to 20 degrees
C. Should be at least 0.055 m radian up to 30 degrees
D. Should be at least 0.055 m radian up to 40 degrees

9. What is the general intact stability criteria requirement on the Area of the GZ curve?
A. Should be at least 0.090 m radian up to 10 degrees
B. Should be at least 0.090 m radian up to 20 degrees
C. Should be at least 0.090 m radian up to 30 degrees
D. Should be at least 0.090 m radian up to 40 degrees
10. What is the general intact stability criteria requirement on the Area of the GZ curve?
A. Should be at least 0.02 m radian between 30 and 40 or between 30 and
angle of down flooding.
B. Should be at least 0.03 m radian between 30 and 40 or between 30 and
angle of down flooding.
C. Should be at least 0.04 m radian between 30 and 40 or between 30 and
angle of down flooding.

@ All Rights Reserved


Issue No. 1 Page 198 of 289
ST. THERESE- MTC COLLEGES SEAMANSHIP 6
Iloilo, Philippines (Advance Trim, Stability and Stress)
STUDENT LEARNING MODULE

E
Revision No. 2 Effectivity date: Reviewed by: Approved by:
GEMMA E. PAGSUGIRON, MBA MA. TERESA S. SARABIA, Ph.D.
01 September 2021 QMR President
D. Should be at least 0.05 m radian between 30 and 40 or between 30 and
angle of down flooding.

Intact Stability Criteria for Container Ship greater than 100 meters

11. What is the intact stability criteria requirement on the initial GM or metacentric
height?
A. Not less than 0.10 meters C. Not more than 0.10 meters
B. Not less than 0.15 meters D. Not more than 0.15 meters
12. What is the intact stability criteria requirement on the righting lever GZ?
A. Shall at least 0.030 meters C. Shall at least 0.032 meters
B. Shall at least 0.031 meters D. Shall at least 0.033 meters
13. What is the intact stability criteria requirement on the righting lever GZ angle of
heel?
A. ≥ 20 degrees C. ≤ 20 degrees
B. ≥ 30 degrees D. ≤ 30 degrees
14. What is the intact stability criteria requirement on the maximum righting lever should
occur?
A. At least 0.042
B. At least 0.043
C. At least 0.044
D. At least 0.045
15. What is the intact stability criteria requirement on the Area under the GZ curve?
A. Not less than 0.009 m radian up to 10 degrees angle of heel
B. Not less than 0.009 m radian up to 20 degrees angle of heel
C. Not less than 0.009 m radian up to 30 degrees angle of heel
D. Not less than 0.009 m radian up to 40 degrees angle of heel
16. What is the intact stability criteria requirement on the Area of the GZ curve?
A. Not less than 0.016 m radian up to 10 degrees angle of heel
B. Not less than 0.016 m radian up to 20 degrees angle of heel
C. Not less than 0.016 m radian up to 30 degrees angle of heel
D. Not less than 0.016 m radian up to 40 degrees angle of heel
17. What is the intact stability criteria requirement on the Area of the GZ curve?
A. Not less than 0.006 m radian between 20 degrees and 30 degrees angle of
heel
B. Not less than 0.006 m radian between 30 degrees and 40 degrees angle of
heel
C. Not less than 0.006 m radian between 40 degrees and 50 degrees angle of
heel
D. Not less than 0.006 m radian between 25 degrees and 35 degrees angle of
heel
18. What is the intact stability criteria requirement on the total area of the GZ curve?

@ All Rights Reserved


Issue No. 1 Page 199 of 289
ST. THERESE- MTC COLLEGES SEAMANSHIP 6
Iloilo, Philippines (Advance Trim, Stability and Stress)
STUDENT LEARNING MODULE

E
Revision No. 2 Effectivity date: Reviewed by: Approved by:
GEMMA E. PAGSUGIRON, MBA MA. TERESA S. SARABIA, Ph.D.
01 September 2021 QMR President
A. Up to the angle of flooding should not be less than 0.027 m radians
B. Up to the angle of flooding should not be less than 0.028 m radians
C. Up to the angle of flooding should not be less than 0.029 m radians
D. Up to the angle of flooding should not be less than 0.030 m radians
19. What is the intact stability criteria requirement on the GM or metacentric height
during the voyage and at all times?
A. Not less than 0.10 meters C. Not more than 0.10 meters
B. Not less than 0.15 meters D. Not more than 0.15 meters
20. What is the intact stability criteria requirement on the maximum righting lever should
occur?
A. Shall at least 0.15 meters C. Shall at least 0.35 meters
B. Shall at least 0.25 meters D. Shall at least 0.40 meters
21. What is the intact stability criteria requirement on the Area under the GZ curve?
A. Not less than 0.04 meters C. Not more than 0.08 meters
B. Not less than 0.06 meters D. Not more than 0.10 meters
22. What is the intact stability criteria requirement on the limiting angle of heel under
action of steady wind?
A. 8 degrees C. 25 degrees
B. 16 degrees D. 30 degrees
23. What is the intact stability criteria requirement on the initial GM or metacentric
height?
A. Not less than 0.10 meters C. Not more than 0.30 meters
B. Not less than 0.20 meters D. Not more than 0.40 meters
24. What is the intact stability criteria requirement on the angle of heel due to shift of
grain?
A. Not be greater than 8 degrees
B. Not be greater than 10 degrees
C. Not be greater than 12 degrees
D. Not be greater than 14 degrees

25. What is the intact stability criteria requirement on the residual area between the
heeling arm curve and righting arm curve?
A. Shall not be less than 0.065 m radians up to 20º angle of heel or angle of
flooding
B. Shall not be less than 0.065 m radians up to 40º angle of heel or angle of
flooding
C. Shall not be less than 0.075 m radians up to 20º angle of heel or angle of
flooding
D. Shall not be less than 0.075 m radians up to 40º angle of heel or angle of
flooding

@ All Rights Reserved


Issue No. 1 Page 200 of 289
ST. THERESE- MTC COLLEGES SEAMANSHIP 6
Iloilo, Philippines (Advance Trim, Stability and Stress)
STUDENT LEARNING MODULE

E
Revision No. 2 Effectivity date: Reviewed by: Approved by:
GEMMA E. PAGSUGIRON, MBA MA. TERESA S. SARABIA, Ph.D.
01 September 2021 QMR President
Oral Assessment (Midterm)

Explain the following and make a video clip wearing upper uniform.

1. Rolling period
2. Inclining test
3. Test correction
4. Stability criteria and requirement

Oral presentation on the chosen research topic directions (this is a video oral presentation):

1. In this activity, you will work a one person only.


2. On a paper, please write down what you think is your idea regarding your chosen
research topic. Please accomplish this task in 3 minutes.
3. When done, please post your output on Power Point Slides and prepare to present.
4. Each person will be given at least 2 minutes to present their outputs.
5. During the presentation, please explain briefly why you think you listed items that
constitute the terms in your chosen research topic.

RUBRICS FOR GRADING ORAL ASSESSMENT

CRITERIA SCORE
Content (40%)
 The contetnt clearly incorporates salient points discussed about the
topic
Creativity and overall presentation (30%)
 The presentation showcases learner’s creativity and originality
Other elements (15%)
 The group effectively use gestures, eye contact, movements, and facial
expressions to convey the message.
 The speaker speaks audibly and clearly.
 The speaker uses props, and costumes appropriately and effectively.
Mastery (15%)
 The pair has mastered the concepts and delivered it well.

Research/Output

Research your OUTPUT for the following topics and explain it:

1. Heeling moment
2. Rolling ship (basic principles)
3. Synchronization
4. Centripetal force

Power Point Presentation, make a 5 minutes presentation of the chosen research topic. The
format of the slides must be:

- Slide 1: Title of the chosen research topic and members

@ All Rights Reserved


Issue No. 1 Page 201 of 289
ST. THERESE- MTC COLLEGES SEAMANSHIP 6
Iloilo, Philippines (Advance Trim, Stability and Stress)
STUDENT LEARNING MODULE

E
Revision No. 2 Effectivity date: Reviewed by: Approved by:
GEMMA E. PAGSUGIRON, MBA MA. TERESA S. SARABIA, Ph.D.
01 September 2021 QMR President
– Slide 2: Justification for choosing the topic
– Slide 3: Existing problems, issues, gaps relevant to the chosen topic
- Font Size and Style: Title: Arial 36-40” Body: 24-28”
- Animation and Design: Free Style
- Follow 7 x 7 rule (7 words and 7 lines in a slide)

RUBRIC FOR GRADING POWER POINT PRESENTATION OF A CHOSEN RESEARCH


TOPIC

CRITERIA SCORE
CONTENT (40%)
 The content clearly incorporates salient points discussed about the
topic.
SLIDE CREATION (15%)
 Presentation flows well and logically. Presentation reflects extensive
use of tools in a creative way.
SLIDE TRANSITIONS (15%)
 Transitions are smooth. Transitions enhance the presentation
PICTURES, CLIP ART BACKGROUND (15%)
 Images are appropriate. Layout is pleasing to the eye.
MECHANICS (15%
 No spelling errors. No grammar errors. Text is in authors’ own
words

RESEARCH WORK

Submit a well-written literature review of your research topic and explain it:

1. Righting moment and Heeling couple


2. Dry docking and grounding (basic requirements)
3. Up thrust
4. Righting moment

The table provided below is a guide in making your literature review. You may add extra
pages.
1. Draft article summary paragraph: 3-4 paragraphs of these will become your short
literature review.
2. Create an order for your paragraph summaries (try this with just 3-4 paragraphs that
relate). You have now created a draft of a short literature review.
3. Revise your short literature review to include paragraph introduction, body, and
conclusions/synthesis.

ARTICLE SUMMARY TABLE (Related Literature and Studies)

1 2
 Reference
 Aim/Objective/Scope

@ All Rights Reserved


Issue No. 1 Page 202 of 289
ST. THERESE- MTC COLLEGES SEAMANSHIP 6
Iloilo, Philippines (Advance Trim, Stability and Stress)
STUDENT LEARNING MODULE

E
Revision No. 2 Effectivity date: Reviewed by: Approved by:
GEMMA E. PAGSUGIRON, MBA MA. TERESA S. SARABIA, Ph.D.
01 September 2021 QMR President
 Participants and sampling
 Context and framework
 Results or findings
 Implications Practice to Research
 Your Comments

Rubric for grading research work on reviewing literature

CRITERIA SCORE
INTRODUCTION (10%)
 Clear overview of paper, demonstrates importance of topic.
BODY (40%)
 Balanced viewpoint: Objective, balanced view from various
perspectives. Coherent theme: Each cited study related to
the topic and to other studies.
 Depth and breadth of research: Variety of studies and
attention to detail about the topic.
 Analysis: Collection of studies analyzed for differences and
commonalities about the topic.
CONCLUSION AND SYNTHESIS (10%)
 Information synthesized and brought to a logical conclusion.
ORGANIZATION AND ALIGNMENT (15%)
 Organization and Alignment: Information logically organized
with good flow. Issues threaded throughout paper.
MECHANICS, APA, REFERENCES (25%)
 Correct spelling, punctuation, sentence structure, word
usage.
 Correct use of APA in body of paper References correctly.
typed, appropriate number and quality

Lesson 11: Rolling Period

Learning Module 11.1: Rolling Period (Week 12, 6 hours)

Competence, Course Outcomes and Learning Outcomes

Competence:

Control trim, stability and stress

Course Outcome:

@ All Rights Reserved


Issue No. 1 Page 203 of 289
ST. THERESE- MTC COLLEGES SEAMANSHIP 6
Iloilo, Philippines (Advance Trim, Stability and Stress)
STUDENT LEARNING MODULE

E
Revision No. 2 Effectivity date: Reviewed by: Approved by:
GEMMA E. PAGSUGIRON, MBA MA. TERESA S. SARABIA, Ph.D.
01 September 2021 QMR President
1. Explain the importance of maintaining stability during loading, unloading
and in-transit in various conditions.
2. Calculate the effect on trim and stability of a ship in the event of damage
to and consequent flooding of a compartment and countermeasures to be
taken.

Learning Outcomes:

At the end of the learning module, the student can:

1. Discuss the factors and conditions of rolling period of a ship.


2. Calculate the following:
– new displacement in tones (W);
– original radius of gyration (K);
– new displacement and new GM (W2 and GM2);
– new mass moment of inertia (I2);
– new radius of gyration (K2);
– new period of roll (T2).

Discussion

11.1 The Period of a Ship

This is the time taken by a ship to roll from one side to the other and back again. When the
period is exactly the same for every roll, the rolling is termed “isochronous”. It is often
assumed that isochronous rolling occurs in every ship for any angle of roll, but this is not
correct. We may accept the following general rules:

a. Different ships have different periods of roll.


b. The same ship will have a different period for different conditions of loading.
c. The same ship will have a longer period when she is tender than when she is stiff.
d. “Winging out” weights will increase the period, all other things being equal.
e. Rolling is isochronous for small angles of roll, up to about ten degrees, but the
period increases slightly for larger angles.

11.2 Synchronism

This is said to occur when the ship’s period of roll is the same as the apparent period of the
waves. When it occurs, the waves gives the ship a “push” each time she rolls, in the
direction in which she is rolling, causing her to roll more and more heavily. In theory, it
would continue until she capsized, but this does not happen in practice because of certain
resistances.

@ All Rights Reserved


Issue No. 1 Page 204 of 289
ST. THERESE- MTC COLLEGES SEAMANSHIP 6
Iloilo, Philippines (Advance Trim, Stability and Stress)
STUDENT LEARNING MODULE

E
Revision No. 2 Effectivity date: Reviewed by: Approved by:
GEMMA E. PAGSUGIRON, MBA MA. TERESA S. SARABIA, Ph.D.
01 September 2021 QMR President
11.3 ‘Still Water’ Rolling

Rolling is a simple harmonic motion. A vessel can be made to roll under wave action. Rolling
period is an important parameter in ship handling too. It is important to be able to measure
the rolling period of a ship.

Figure 11.1

The stop watch is started when the roll has reached the maximum heel on any side, port
side or starboard side. The watch is stopped after the vessel rolls to the opposite side and
then returns to the maximum heel on the side from which time measurement was started.

The rolling characteristics of a ship are governed by:


1. the GM and;
2. the distribution of the weight components of the ship’s structure and deadweight
items with respect to the rolling axis, which is assumed to be at the center of gravity
of the ship (G).

The roll period (T) in seconds is the time taken for the ship to complete one complete
oscillation i.e. the time it takes for the ship to roll from one side back through the upright to
the extent of it’s roll on the other side and back again.

One complete oscillation

Roll Period (T) in seconds

Figure 11.2: Ship rolling

The amplitude on the roll is defined as the extent of the roll in degrees.
11.4 Unrestricted Rolling
Unrestricted rolling is a hypothetical condition that assumes the ship is rolling in a
‘frictionless’ fluid such that the amplitude of the oscillations remains constant. In reality the

@ All Rights Reserved


Issue No. 1 Page 205 of 289
ST. THERESE- MTC COLLEGES SEAMANSHIP 6
Iloilo, Philippines (Advance Trim, Stability and Stress)
STUDENT LEARNING MODULE

E
Revision No. 2 Effectivity date: Reviewed by: Approved by:
GEMMA E. PAGSUGIRON, MBA MA. TERESA S. SARABIA, Ph.D.
01 September 2021 QMR President
amplitude of one roll decreases with each half-roll cycle until eventually the ship settles in
the upright condition in still water (being due to water and air resistance).

The energy in this assumed un-damped roll is totally potential energy at the limit of the roll,
and totally kinetic energy the instant the ship passes through the upright position. At any
intermediate position within the roll cycle the energy will be partly kinetic and partly
potential; but the sum of these two quantities will remain constant. The ship will act in a
similar manner to a pendulum under the same conditions.

Figure 11.3

At the extent of the roll, (A) and (C) in Figure 11.3, the ship will possess all potential energy
(PE) that will be equivalent to: PE at extent of roll to θ ° = Area under righting moment
curve from 0° to θ ° .

As the ship passes through the upright position, (B) in Figure 11.3, the GZ and hence
righting moment will be zero and potential energy at this point in the roll will be zero.
However, the ship will possess all kinetic energy by virtue of its motion as it passes through
the upright position.

Under the circumstance described the roll period (T) is approximated by the formula:

T (secs) =

where: ‘I’ is the moment of inertia of the ship’s structure and all deadweight components
about the rolling axis (G);
‘W’ is the ship’s displacement in tones;
‘g’ is the acceleration due to gravity (9.81 m/s2); and
‘GM’ is the ship’s effective metacentric height.

The value of I is found by: I = Σwr2

where ‘w’ is the weight of each structural component of the ship and each item of
deadweight and ‘r’ is the distance that each component is from the rolling axis, assumed to
be at G.

@ All Rights Reserved


Issue No. 1 Page 206 of 289
ST. THERESE- MTC COLLEGES SEAMANSHIP 6
Iloilo, Philippines (Advance Trim, Stability and Stress)
STUDENT LEARNING MODULE

E
Revision No. 2 Effectivity date: Reviewed by: Approved by:
GEMMA E. PAGSUGIRON, MBA MA. TERESA S. SARABIA, Ph.D.
01 September 2021 QMR President
2 2
The quantity Σwr can be written as Wk where:

W is the ship’s displacement and


k is the radius of gyration.

11.5 Radius of Gyration

The radius of gyration might be a difficult concept to understand but may be loosely defined
as being for a ship the distance from the center of gravity (or rolling axis) at which the total
weight (W) would have to be concentrated in order to give the ship the same moment of
inertia as it actually has.

For any particular ship the radius of gyration can be changed by altering the distribution of
deadweight about the rolling axis

Figure 11.4

Moving weight to the sides of the ship away from the rolling axis (G) increases the radius of
gyration (k) and increases the ship’s rolling period.

11.6 Rolling Period Formula

Because I = Σwr2, the formula for the still water rolling period becomes:

T (secs) =

which simplifies to:


2 πk
T (secs) =
√ g xGM
This formula is substantially correct for small angles of heel where the formula GZ = GM x
Sin θ is valid.

The formula shows that for a constant value of I (or k) the rolling period is inversely
proportional to the square root of the GM (as long as GM is positive).

As GM increases the rolling period reduces.

The formula also shows that the rolling period is directly proportional to the radius of
gyration (k), that is, ask increases (by ‘winging out’ weights say), the rolling periods
increases.

@ All Rights Reserved


Issue No. 1 Page 207 of 289
ST. THERESE- MTC COLLEGES SEAMANSHIP 6
Iloilo, Philippines (Advance Trim, Stability and Stress)
STUDENT LEARNING MODULE

E
Revision No. 2 Effectivity date: Reviewed by: Approved by:
GEMMA E. PAGSUGIRON, MBA MA. TERESA S. SARABIA, Ph.D.
01 September 2021 QMR President

Process me
Example 1

Calculate the natural rolling period of a ship for which k is 4.5 m and GM is 1.15 m.
Solution:
2 πk
T (secs) = where: ‘g’ is the acceleration due to gravity 9.81 m/s 2
√ g xGM
2 π x 4.5
=
√ 9.81 x 1.15
28.274
=
√ 11.2815
28.274
T (secs) = = 8.4 seconds
3.36

Example 2

A ship displaces 15,500 tonnes, has GM 1.25 m and a roll period of 12 seconds. The ship
loads 150 tonnes in position 6.3 m above G.

2 πk
Formula: T (secs) =
√ g xGM
Solution:
1. Calculate the initial radius of gyration (k).

T x √ g x GM 12 x √ 9.81 x 1.25 12 x √ 12.2625


k= = ==
2π 2π 2π
12 x 3.502
k=

42.024 42.024
k= = = 6.688 m
2π 6.2832
2. Calculate the new value of I and new GM:
Original I = Wk2 = W x K2

I = Wk2 = 15,500 x 6.6882 = 693,305 m2


I value of added weight = wd2 = 150 x 6.32 = + 5,954 m2
Total = 699,259 m2

@ All Rights Reserved


Issue No. 1 Page 208 of 289
ST. THERESE- MTC COLLEGES SEAMANSHIP 6
Iloilo, Philippines (Advance Trim, Stability and Stress)
STUDENT LEARNING MODULE

E
Revision No. 2 Effectivity date: Reviewed by: Approved by:
GEMMA E. PAGSUGIRON, MBA MA. TERESA S. SARABIA, Ph.D.
01 September 2021 QMR President
wxd 150 t x 6.3 m 945
GGV = = = = 0.0604 m
W +w 15500t +150 t 15650

Correction to I for shift of G = W x (GGV)2 = 15,650 m x 0.06042 = 57 m4


therefore:
Find the value of I about the new G = 699,259 m4 – 57m4 = 699,202 m4

New GM = Initial GM – GGV


New GM = 1.25 m – 0.0604 t-m = 1.190 m

3. Calculate the new value of k:


4
I
I = Wk2∴ k2 = = 699,202m = 44.677 m
W 15650 t
New k = √ 44.677
New k =6.684

4. Calculate the new roll period:


2 πk
New T (secs) =
√ g xGM
2 π x 6.684 41.997 41.997
= = =
√ 9.81 x 1.190 √ 11.674 3.147
New T =13.3 seconds = 13 seconds

Example 3

A ship of 10,500 tonnes displacement has GM 0.50 m. The period of roll in still water is 20
seconds. Calculate the new period of roll if a mass of 95 tonnes is discharged from a
position 16 m above the center of gravity.

Procedures:
1. Calculate the new displacement in tonnes
2. Estimate the original radius of gyration (K)
3. Evaluate the new displacement and new GM (W 2 and GM2)
4. Calculate the new mass moment of inertia (I2)

@ All Rights Reserved


Issue No. 1 Page 209 of 289
ST. THERESE- MTC COLLEGES SEAMANSHIP 6
Iloilo, Philippines (Advance Trim, Stability and Stress)
STUDENT LEARNING MODULE

E
Revision No. 2 Effectivity date: Reviewed by: Approved by:
GEMMA E. PAGSUGIRON, MBA MA. TERESA S. SARABIA, Ph.D.
01 September 2021 QMR President
5. Calculate the new radius of gyration (K2)
6. Evaluate the new period of roll (T2)

Solution:
1. Calculate the new displacement in tones (W2)

W2 = WO – w
W2 = 10,500 tonnes – 95 tonnes
W2 = 10,405 tonnes

2. Estimate the initial/original radius of gyration (k).

T x √ g x GM 20 x √ 9.81 x 0.50 20 x √ 4.905


k= = =
2π 2π 2π
20 x 2.215
k=

44.3 44.3
k= = = 7.051 m
2 π 6.2832

3. Calculate the new value of I and new GM:

Original I = Wk2 = W2 x K2

IO = Wk2 = 10,405 x 7.0512 = 517,301 m2


New I value of discharged weight = wd2 = 95 x 162 = - 24,320 m2
Total = 429,981 m2
wxd 95 t x 16 m 1520t−m
GGV =
W +w = 10500t−95t = 10,405 t = 0.146 m

Correction to I for shift of G = W x (GGV)2 = 10,405 m x 0.1462 = 222 m4

Therefore:
Calculate final value of I about the new G = 429,981 m4 – 222 m4 = 429,759 m4

New GM (GM2) = Initial GM + GGV


New GM (GM2) = 0.50 m + 0.146 m =0.646 m

4. Calculate the new value of radius of gyration (k2):

4
I
I1= Wk2∴k2 = = 429,981 m = 41.32 m
W 10,405t

@ All Rights Reserved


Issue No. 1 Page 210 of 289
ST. THERESE- MTC COLLEGES SEAMANSHIP 6
Iloilo, Philippines (Advance Trim, Stability and Stress)
STUDENT LEARNING MODULE

E
Revision No. 2 Effectivity date: Reviewed by: Approved by:
GEMMA E. PAGSUGIRON, MBA MA. TERESA S. SARABIA, Ph.D.
01 September 2021 QMR President
New k (k2) = √ 41.32
New k (k2) = 6.43 m

5. Evaluate the new period of roll (T2):


2 πk
New T (secs) =
√ g x GM
2 π x 6.43 40.4
= =
√ 9.81 x 0.646 √ 6.337
40.4
=
2.517
New T =16.05 seconds = 16 seconds

Do this

Activity 1 – Rolling Period Problems

Calculate the natural rolling period of a ship when the radius of gyration is 6.5 m and
metacentric height is 0.70 m.

Do this

Activity 2 – Rolling Period Problems

A ship displaces 15,500 tonnes, has GM 1.35 m and a roll period of 16 seconds. The ship
loads 80 tonnes in position 6.5 m above G. Calculate the following:

@ All Rights Reserved


Issue No. 1 Page 211 of 289
ST. THERESE- MTC COLLEGES SEAMANSHIP 6
Iloilo, Philippines (Advance Trim, Stability and Stress)
STUDENT LEARNING MODULE

E
Revision No. 2 Effectivity date: Reviewed by: Approved by:
GEMMA E. PAGSUGIRON, MBA MA. TERESA S. SARABIA, Ph.D.
01 September 2021 QMR President
1. Initial radius of gyration (k);
2. New value of I;
3. New GM:
4. New value of k:
5. New roll period:

Assessment – Calculation of Rolling Period

A ship of 20,000 tonnes displacement has GM 0.75 m. The period of roll in still
water is 18 seconds. Calculate the new period of roll if a mass of 150 tonnes is discharged
from a position 18 m above the center of gravity. Calculate the following:
1. New displacement in tonnes (W2);
2. Initial radius of gyration (K);
3. New Moment of Inertia (I);
4. New GM;
5. New radius of gyration (K2);
6. New Period of Roll (T2).

@ All Rights Reserved

You might also like